Chapter 13
COMPLICATIONS OF PREGNANCY, CHILDBIRTH, AND THE PUERPERIUM
Introduction
Codes for complications of pregnancy, childbirth, and the puerperium are found in Chapter 13. There are no fifth digit subclassifications that identify the episode of care. Instead of episode of care designations, the majority of codes for complications that occur during pregnancy are organized to identify the trimester, which is captured by the fourth, fifth, or sixth character. In order to assign the most specific code, documentation will now need to provide the trimester or the weeks of gestation. The fourth, fifth, or sixth character may also capture the stage of pregnancy for complications that can occur at any point or at certain times in the pregnancy—during second or third trimester, childbirth, or postpartum, such as eclampsia in category O15. Some complications, specifically those that typically occur or are treated only in a single trimester, such as ectopic pregnancy (O00), do not identify the trimester. In addition, complications that occur only during childbirth or the puerperium contain that information in the code description, such as obstructed labor due to generally contracted pelvis (O65.1) or puerperal sepsis (O85).
Another key point in assigning pregnancy codes is the need for a 7th character identifying the fetus for which the complication code applies. For single gestation, or when the documentation is insufficient to identify the fetus, the 7th character ‘0’ for not applicable/unspecified is assigned. For multiple gestations, each fetus should be identified with a number as fetus 1, fetus 2, fetus 3, etc. The fetus or fetuses affected by the condition should then be clearly identified using the number assigned to the fetus. For example, a triplet gestation in the third trimester with fetus 1 having no complications, fetus 2 in a separate amniotic sac having polyhydramnios, and fetus 3 having hydrocephalus with maternal pelvic disproportion would require reporting of the complications as follows: Fetus 1 – No codes; Fetus 2 – O40.3XX2 Polyhydramnios, third trimester, fetus 2; Fetus 3 – O33.6XX3 Maternal care for disproportion due to hydrocephalic fetus, fetus 3. An additional code identifying the triplet pregnancy would also be reported.
Coding Instructions
There are important coding notes and instructions at the chapter level. The first note indicates that codes from Chapter 15 are to be used only on maternal records, never on newborn records. Codes that capture conditions in the newborn that were caused by a maternal condition are found in Chapter 16 Certain Conditions Originating in the Perinatal Period, in code categories P00-P04, and these codes are reported only on the newborn record.
Codes in Chapter 15 are to be used for conditions related to or aggravated by the pregnancy, childbirth, or the puerperium, which may be either maternal causes or obstetric causes. While not specifically addressed in the chapter level instructions, it should also be noted that codes in Chapter 15 are sequenced first over codes from other chapters. Codes from other chapters are reported additionally following any instructions at the code level. For example, codes for diabetes mellitus in pregnancy, childbirth, and the puerperium (O24) are sequenced first followed by a code from categories E08-E13 to identify the specific type of diabetes and any complications or manifestations.
Because most complications related to pregnancy require identification of the trimester, there is also an explanation on how each trimester is calculated. Trimester is calculated using the first day of the last menstrual period. Calculations for trimesters are as follows:
First trimester – less than 14 weeks 0 days
Second trimester – 14 weeks 0 days to less than 28 weeks 0 days
Third trimester – 28 weeks 0 days to delivery
There is also a note to use an additional code to identify weeks of gestation. Weeks of gestation codes are supplementary codes found in category Z3A.
Chapter Organization
The chapter is organized with categories for ectopic pregnancy, molar pregnancy, and pregnancy with abortive outcome listed first. Categories for complications primarily related to pregnancy are listed next, followed by codes related to labor and delivery, which includes both complications of labor and delivery and normal delivery. Complications related to the puerperium are the final listed codes. There are some exceptions. Conditions that have the potential to complicate pregnancy, childbirth, and/or the puerperium, such as edema, proteinuria, and hypertensive disorders (O10-O16) are listed together with fourth, fifth, or sixth characters that identify the stage of care. Codes for supervision of high risk pregnancy (O09) and abnormal findings on antenatal screening of the mother (O28) are also found within this chapter.
Multiple gestation placenta status is captured with a combination code for multiple gestation that includes information on the number of placentas and amniotic sacs (O30). Inconclusive fetal viability (O36.80) is also listed in Chapter 15 as are other codes for maternal care for other fetal problems. Refer to the table below for a complete list of the blocks of codes in ICD-10-CM.
ICD-10-CM Blocks
O00-O08
Pregnancy with Abortive Outcome
O09
Supervision of High Risk Pregnancy
O10-O16
Edema, Proteinuria and Hypertensive Disorders in Pregnancy, Childbirth and the Puerperium
O20-O29
Other Maternal Disorders Predominantly Related to Pregnancy
O30-O48
Maternal Care Related to the Fetus and Amniotic Cavity and Possible Delivery Problems
O60-O77
Complications of Labor and Delivery
O80-O82
Encounter for Delivery
O85-O92
Complications Predominantly Related to the Puerperium
O94-O9A
Other Obstetric Conditions, Not Elsewhere Classified
Exclusions
In ICD-10-CM Chapter 15, the excludes notes are as follows:
Excludes1
Excludes2
Supervision of normal pregnancy (Z34.-)
Mental and behavioral disorders associated with the puerperium (F53)
Obstetrical tetanus (A34)
Postpartum necrosis of pituitary gland (E23.0)
Puerperal osteomalacia (M83.0)
Supervision of normal pregnancy is a type 1 excludes note which means that codes in category Z34 cannot be reported in conjunction with codes from Chapter 15. Codes for supervision of normal pregnancy are used only for patients who have no conditions complicating maternal or obstetric care. Codes in category Z34 are sub-classified as normal first pregnancy (Z34.0), other normal pregnancy (Z34.8), and normal pregnancy unspecified (Z34.9). A fifth character is required to identify the trimester.
The Excludes2 note for mental and behavioral disorders associated with the puerperium refers to postpartum depression reported with code F53.0, and puerperal psychosis reported with F53.1. Because puerperal psychosis and postpartum depression are listed as a type 2 excludes note, codes F53.0 and F53.1 can be reported with other complications of the puerperium when the postpartum depression or psychosis occur together with other complications.
Chapter Guidelines
Chapter 15 guidelines include information covering general rules and sequencing of codes as well as coding rules for specific conditions. The guideline categories are as follows:
General rules for coding obstetric cases
Selection of OB principle diagnosis
Pre-existing conditions versus conditions due to the pregnancy
Pre-existing hypertension in pregnancy
Fetal conditions affecting the management of the mother
HIV infection in pregnancy, childbirth, and the puerperium
Diabetes mellitus in pregnancy
Long term use of insulin
Gestational (pregnancy-induced) diabetes
Sepsis and septic shock complicating abortion, pregnancy, childbirth, and the puerperium
Puerperal sepsis
Alcohol, tobacco, and drug use during pregnancy, childbirth, and the puerperium
Poisoning, toxic effects, adverse effects, and underdosing in a pregnant patient
Normal delivery (O80)
The peripartum and postpartum periods
Sequelae of complications of pregnancy, childbirth, and the puerperium (O94)
Termination of pregnancy and spontaneous abortions
Abuse in a pregnant patient
General Rules for Coding Obstetric Cases
General rules cover sequencing priority, assigning a character for the trimester and special coding rules related to the trimester, and assigning a 7th character to identify the fetus.
Sequencing Priority
Codes in Chapter 15 Complications of Pregnancy, Childbirth, and Puerperium (O00-O9A) have sequencing priority over codes from other chapters
Additional codes from other chapters may be reported with Chapter 15 codes to specify the condition further
Documentation that the pregnancy is incidental to the encounter is reported with code Z33.1 Pregnant state, incidental. It is the provider’s responsibility to document that the condition being treated is not affecting the pregnancy
Use Chapter 15 Codes Only on the Maternal Record
Use only on maternal records, never on newborn records
Codes that capture conditions in the newborn that were caused by a maternal condition are found in Chapter 16 Certain Conditions Originating in the Perinatal Period, in code categories P00-P04, and these codes are reported only on the newborn record
Final Character for Identification of Trimester
Most codes in Chapter 15 have a final character indicating the trimester
Assignment of the final character for trimester is based on the provider’s documentation which may identify the trimester or the number of weeks gestation for the current encounter
Trimesters are calculated using the first day of the last menstrual period as follows:
First trimester – less than 14 weeks 0 days
Second trimester – 14 weeks 0 days to less than 28 weeks 0 days
Third trimester – 28 weeks 0 days to delivery
The final character identifying the trimester may be the 4th, 5th, or 6th character
Codes that do not identify the trimester include:
Conditions that only occur during a single specific trimester
Conditions for which the concept of trimester does not apply
Conditions may have characters only for certain trimesters because the condition does not occur in all trimesters but does occur in more than one trimester. For example, category O14 Pre-eclampsia, has specific codes only for the second and third trimesters because pre-eclampsia does not occur in the first trimester (except in patients with pre-existing hypertension, and this condition is reported with codes from category O11)
If a delivery occurs during the admission, and there is an “in childbirth” option for the complication, the code for “in childbirth” is assigned
Trimester Selection for Inpatient Admissions Encompassing More Than One Trimester
When an inpatient admission encompasses more than one trimester, the code is assigned based on when the condition developed- not when the discharge occurred (e.g. if the condition developed during the second trimester and the patient was discharged during the third trimester, the code for the second trimester is assigned)
If the condition being treated developed prior to the current admission/encounter, or was a pre-existing condition, the trimester character at the time of the admission/encounter is used
Unspecified Trimester
Unspecified trimester character should be used only when the documentation is insufficient to determine the trimester and it is not possible to obtain clarification from the provider
7th Character for Fetus Identification
Some complications of pregnancy and childbirth occur more frequently in multiple gestation pregnancies. These complications may affect one or more fetuses and require a 7th character to identify the fetus or fetuses affected by the complication.
The following categories/subcategories require identification of the fetus:
O31 Complications specific to multiple gestation
O32 Maternal care for malpresentation of fetus
O33.3 Maternal care for disproportion due to outlet contraction of pelvis
O33.4 Maternal care for disproportion of mixed maternal and fetal origin
O33.5 Maternal care for disproportion due to unusually large fetus
O33.6 Maternal care for disproportion due to hydrocephalic fetus
O35 Maternal care for known or suspected fetal abnormality and damage
O36 Maternal care for other fetal problems
O40 Polyhydramnios
O41 Other disorders of amniotic fluid and membranes
O60.1 Preterm labor with preterm delivery
O60.2 Term delivery with preterm labor
O64 Obstructed labor due to malposition and malpresentation of fetus
O69 Labor and delivery complicated by umbilical cord complications
The 7th character identifies the fetus for which the complication code applies
For multiple gestations, each fetus should be identified with a number as Fetus 1, Fetus 2, Fetus 3, etc. The fetus or fetuses affected by the condition should be documented using the number assigned to the fetus. The complication code is then assigned for each fetus affected by the complication
The 7th character ‘0’ for not applicable/unspecified is assigned in cases of single gestations, when the documentation is insufficient to identify the fetus affected and it is not possible to obtain clarification, or when it is not possible to determine clinically which fetus is affected
Selection of OB Principle or First-Listed Diagnosis
These guidelines relate to assignment and sequencing of codes for routine outpatient prenatal visits, prenatal visits for high-risk patients, and selection of principle or first-listed diagnosis for episodes of care when no delivery occurs, when delivery occurs, and for the outcome of delivery.
Routine Outpatient Prenatal Visits
A code from category Z34 Encounter for supervision of normal pregnancy, should be used as the first listed diagnosis when there are no documented complications present
A code from category Z34 should not be used with another code from Chapter 15
Supervision of high risk pregnancy codes in category O09 are for use only during the prenatal period. For complications that occur during the labor and delivery episode as a result of a high risk pregnancy, assign the applicable complication code(s) from Chapter 15
Assign code O80 Encounter for full-term uncomplicated delivery if there are no complications during labor and delivery
Supervision of High Risk Pregnancy
A code from category O09 Supervision of high-risk pregnancy, should be assigned as the first-listed diagnosis for routine prenatal outpatient visits for patients with high-risk pregnancies
Secondary codes from Chapter 15 may be used in conjunction with codes from category O09
Episodes When No Delivery Occurs
The principle diagnosis should correspond to the principle complication of the pregnancy which necessitated the encounter as documented by the provider
When more than one documented complication exists, and all are treated or monitored, any of the complications may be sequenced first
Episodes When a Delivery Occurs
When an obstetric patient delivers during an admission, the condition prompting the admission is sequenced as the principal diagnosis
If multiple conditions prompted the admission, the condition most related to the delivery is listed as the principal diagnosis. A code for any complication of the delivery should be assigned as an additional diagnosis
If the patient was admitted with a condition that resulted in the performance of a cesarean procedure, that condition should be reported as the principal diagnosis
If the reason for the admission/encounter was unrelated to the condition resulting in the cesarean delivery, the condition related to the reason for the admission/encounter should be selected as the principal diagnosis
Outcome of Delivery
A code from category Z37 Outcome of delivery should be included on every maternal record when a delivery has occurred
Codes from category Z37 are not used on subsequent maternal records
Codes from category Z37 are not used on the newborn record
Pre-Existing Conditions Vs. Conditions Due to the Pregnancy
Certain categories in Chapter 15 distinguish between conditions of the mother that existed prior to pregnancy (pre-existing) and those that are a direct result of the pregnancy. Two examples are hypertension (O10, O11, O13) and diabetes mellitus (O24). The physician must provide clear documentation as to whether the condition existed prior to pregnancy or whether it developed during the pregnancy or as a result of the pregnancy.
Categories that do not distinguish between pre-existing conditions and pregnancy-related conditions may be used for either.
If a puerperal complication develops postpartum during the delivery encounter, and a specific code for the puerperal complication exists, it is acceptable to report the code for the puerperal complication with codes related to complications of pregnancy and childbirth.
Pre-Existing Hypertension in Pregnancy
Category O10 Pre-existing hypertension complicating pregnancy, childbirth and the puerperium includes codes for hypertensive heart and hypertensive chronic kidney disease. When one of these codes is assigned, it is necessary to add a secondary code from the appropriate hypertension category (I11-I13) to identify the type of hypertensive heart and/or chronic kidney disease.
Fetal Conditions Affecting the Management of the Mother
Two categories are available for reporting fetal conditions affecting the mother—O35 Maternal care for known or suspected fetal abnormality and damage; and O36 Maternal care for other fetal problems. Guidelines for assigning these codes are as follows:
Assign a code from categories O35 or O36 only when the fetal condition is actually responsible for modifying the management of the mother. This may include:
Diagnostic studies
Additional observation
Special care
Termination of the pregnancy
In Utero Surgery
When surgery is performed on the fetus, a diagnosis code from category O35 should be assigned to identify the fetal condition
Codes from Chapter 16 Certain Conditions Originating in the Perinatal Period are not used on the maternal record to identify fetal conditions for in utero surgery. Surgery performed in utero on a fetus is still coded as an obstetric encounter
HIV Infection in Pregnancy
During pregnancy, childbirth, and the puerperium, HIV infection is reported as follows:
A patient admitted because of an HIV-related illness should receive a principal diagnosis from subcategory O98.7-HIV disease complicating pregnancy, childbirth, and the puerperium followed by the code(s) for the HIV related illness(es) [B20 for symptomatic HIV disease/AIDs reported additionally along with specific codes identifying the HIV-related illness]
Patients admitted with asymptomatic HIV infection status during pregnancy, childbirth, and the puerperium should receive codes from subcategories O98.7- and Z21 Asymptomatic HIV infection status
Diabetes Mellitus in Pregnancy
Diabetes mellitus is a significant complicating factor in pregnancy. All pregnant women with diabetes mellitus are considered to have a complication that affects maternal and obstetric care, so a code from category O24 Diabetes mellitus in pregnancy, childbirth, and the puerperium should be assigned for outpatient and inpatient encounters with one or more codes from categories E08-E13 to provide additional information about the type of diabetes and any specific complications and/or manifestations. Coding and sequencing are as follows:
A code from category O24 is reported first
A code or codes from the appropriate category (E08-E13) in Chapter 4 is assigned to identify specific complications/manifestations
Long Term Use of Insulin
If the diabetes is being treated with insulin or oral medications, assign code Z79.4 Long-term (current) use of insulin or code Z79.84 Long-term (current) use of oral hypoglycemic drugs.
If the patient is treated with both oral medications and insulin, only the code for insulin-controlled should be assigned
Gestational (Pregnancy-Induced) Diabetes
Gestational diabetes can occur during the second and third trimesters in women who were not diabetic prior to pregnancy. Gestational diabetes can cause complications similar to those in patients with pre-existing diabetes. It also puts the woman at risk for developing diabetes after the pregnancy. Coding guidelines for reporting gestational diabetes are as follows:
Assign a code from subcategory O24.4 Gestational diabetes mellitus
Do not assign any other codes from category O24 Diabetes mellitus in pregnancy, childbirth and the puerperium in conjunction with a code from O24.4-.
The provider must document whether the gestational diabetes is being controlled by diet, oral hypoglycemics, or insulin
If documentation indicates the gestational diabetes is being controlled with both diet and insulin, report only the code for insulin-controlled
If documentation indicates the gestational diabetes is being treated with both diet and oral hypoglycemic medications, report only the code for “controlled by oral hypoglycemic drugs”.
Do not assign code Z79.4 Long-term (current) use of insulin or code Z79.84 Long-term (current) use of oral hypoglycemic drugs with codes from subcategory O24.4
An abnormal glucose tolerance test in pregnancy without specific documentation by the provider that the patient has gestational diabetes is assigned a code from subcategory O99.81 Abnormal glucose complicating pregnancy, childbirth, and the puerperium
Sepsis and Septic Shock Complicating Abortion, Pregnancy, Childbirth, and the Puerperium
There are specific codes for sepsis and septic shock complicating abortion, pregnancy, childbirth, and the puerperium in Chapter 15. When one of these codes is assigned, a code for the specific type of infection should be assigned as an additional diagnosis. When severe sepsis is present, a code from subcategory R65.2- Severe sepsis should be used along with codes for any documented organ dysfunction associated with the sepsis as additional diagnoses.
Puerperal Sepsis
Code O85 Puerperal sepsis should be assigned with a secondary code to identify the causal organism:
For a bacterial infection, assign a code from categories B95-B96
Do not assign a code from category A40 Streptococcal sepsis, or A41 Other sepsis
If applicable, use an additional code (R65.2-) to identify severe sepsis
Assign additional codes for any documented acute organ dysfunction associated with the sepsis
Do not assign code O85 for sepsis following an obstetrical procedure
Alcohol, Tobacco, and Drug Use During Pregnancy, Childbirth, and the Puerperium
Alcohol use during pregnancy, childbirth, and the puerperium
For documented alcohol use during the pregnancy or postpartum, assign a code from subcategory O99.31 Alcohol use complicating pregnancy, childbirth, and the puerperium
Assign a secondary code from category F10 Alcohol related disorders, to identify manifestations of the alcohol use
Tobacco use during pregnancy, childbirth, and the puerperium
For documented use of any type of tobacco product during the pregnancy or postpartum, assign a code from subcategory O99.33 Smoking (tobacco) complicating pregnancy, childbirth, and the puerperium
Assign a secondary code from category F17 Nicotine dependence, or code Z72.0 Tobacco use to identify the type of nicotine dependence
Drug use during pregnancy, childbirth, and the puerperium
Codes from subcategory O99.32 Drug use complicating pregnancy, childbirth, and the puerperium should be assigned for any case in which the mother uses drugs during the pregnancy or postpartum period. This may involve illegal drugs or the inappropriate use or misuse of prescription drugs
Assign secondary codes from categories F11-F16 and F18-F19 to identify manifestations of the drug use
Poisoning, Toxic Effects, Adverse Effects, and Underdosing in a Pregnant Patient
Sequencing rules for poisoning, toxic effects, adverse effects, and underdosing in pregnancy are as follows:
A code from subcategory O9A.2 Injury, poisoning and certain other consequences of external causes complicating pregnancy, childbirth, and the puerperium is sequenced first
The appropriate code for the injury, poisoning, toxic effect, adverse effect, or underdosing is sequenced second
Additional codes are assigned to specify the condition(s) caused by the poisoning, toxic effect, adverse effect, or underdosing
Normal Delivery
Code O80 is assigned for a normal uncomplicated delivery. Guidelines are as follows:
Encounter for Full Term Uncomplicated Delivery
Use code O80 for a full term uncomplicated delivery of a single, healthy infant without any antepartum, delivery, or postpartum complications during the delivery episode
Code O80 is always assigned as the principal diagnosis
Do not use code O80 if any other code from Chapter 15 is needed to describe a current complication of the antenatal, delivery, or perinatal period
Additional codes from other chapters may be assigned with code O80 if the condition the code describes is not related to, or in any way associated with a complication of the pregnancy
Uncomplicated Delivery with Resolved Antepartum Complication
Code O80 may be used if the patient had a complication at some point during the pregnancy, as long as the condition is not present at the time of admission for delivery
Outcome of Delivery for O80
The only valid outcome of delivery code appropriate for use with code O80 is Z37.0 Single live birth
The Peripartum and Postpartum Periods
Peripartum is defined as the last month of pregnancy to five months postdelivery. Postpartum is defined as beginning immediately after delivery and continues for six weeks following delivery. Guidelines related to the peripartum and postpartum periods are as follows:
Complications
A postpartum complication is any complication occurring within the 6-week period following delivery
Pregnancy-related complications after the 6-week period may be reported with codes from Chapter 15 if the provider documents that the condition is pregnancy-related
Pregnancy-associated cardiomyopathy, code O90.3, is unique in that it may be diagnosed in the third trimester, but may continue to progress for months after delivery. For this reason, it is considered a peripartum complication
For documented pregnancy-associated cardiomyopathy, assign code O90.3 Peripartum cardiomyopathy only when the condition develops as a result of pregnancy in a patient who did not have pre-existing heart disease
Admission for Routine Postpartum Care following Delivery Outside Hospital
Assign code Z39.0 Encounter for care and examination of mother immediately after delivery, when the mother delivers outside the hospital and is admitted only for routine postpartum care without any documented complications
Sequelae of Complication of Pregnancy, Childbirth, and the Puerperium
Code O94 Sequela of complication of pregnancy, childbirth, and the puerperium is assigned in cases when an initial complication of pregnancy develops sequelae that require care or treatment at a future date. Coding guidelines are as follows:
Code O94 may be used at any time after the initial postpartum period
Code O94 is sequenced following the code that describes the sequela of the complication
Termination of Pregnancy and Spontaneous Abortions
Abortion with liveborn fetus
When an attempted termination of pregnancy results in a liveborn fetus, assign code Z33.2 Encounter for elective termination of pregnancy
Assign a code from category Z37 Outcome of delivery as a secondary diagnosis
Retained Products of Conception Following an Abortion
Assign code O03.4 Incomplete spontaneous abortion without complication or O07.4 Failed attempted termination of pregnancy without complication for subsequent encounters for retained products of conception following a spontaneous abortion or elective termination of pregnancy without complications
Use these codes for subsequent encounters for retained products of conception even if the patient was discharged previously with a discharge diagnosis of complete abortion
If the patient has a specific complication associated with the spontaneous abortion or elective termination of pregnancy in addition to retained products of conception, assign the appropriate complication in category O03 or O07 instead of code O03.4 or O07.4
Complications Leading to Abortion
Chapter 15 codes may be used as additional codes to identify any documented complications of the pregnancy in conjunction with codes in categories O04 Complications following (induced) termination of pregnancy, O07 Failed attempted termination of pregnancy or O08 Complications following ectopic and molar pregnancy
Abuse in a Pregnant Patient
Suspected or confirmed abuse of a pregnant patient is reported as follows:
A code from one of the following subcategories in Chapter 15 is sequenced first:
O9A.3 Physical abuse complicating pregnancy, childbirth, and the puerperium
O9A.4 Sexual abuse complicating pregnancy, childbirth, and the puerperium
O9A.5 Psychological abuse complicating pregnancy, childbirth, and the puerperium
Assign secondary codes for any associated current injury resulting from the abuse
General Documentation Requirements
There are a number of general documentation requirements that are briefly reviewed here. They will be addressed in more detail in subsequent sections, and specific examples of the required documentation will be provided in the coding scenarios.
Trimester
Most codes for complications of pregnancy require identification of the trimester. The provider may document either the trimester or the weeks of gestation for complications occurring during pregnancy. If the weeks of pregnancy are documented, the coder must identify the trimester based on the ICD-10-CM definitions. This information is provided in coding note instructions at the beginning of Chapter 15. In addition, a supplementary code from category Z3A Weeks of gestation should be assigned to identify the specific week of gestation when documented.
Fetus
Some complications of pregnancy and childbirth occur more frequently in multiple gestation pregnancies than in single gestation pregnancies and these complications may affect one or more of the fetuses. To address this, 14 code categories and subcategories in ICD-10-CM now require identification of the fetus affected by the complication. When documenting these complications in multiple gestation pregnancies, the provider will clearly need to identify every fetus affected by each identified complication by assigning a number to the fetus. There are 7th character extensions that specifically identify each fetus up to five as Fetus 1, Fetus 2, Fetus 3, Fetus 4, and Fetus 5. For multiple gestations with more than five fetuses, the remaining fetuses are identified as “other fetus” by the seventh character extension 9. For single gestations, or multiple gestations in which the affected fetus has not been identified, the 7th character extension 0 is used to indicate that the fetus designation is not applicable or unspecified.
Combination Codes
Another documentation requirement relates to category O30 Multiple gestation. The code identifying the twin, triplet, quadruplet, or other specified multiple gestation includes the multiple gestation placenta status. These combination codes require documentation of the number of placentas and amniotic sacs for twin pregnancies as follows:
O30.01- Twin pregnancy, monochorionic, monoamniotic
O30.03- Twin pregnancy, monochorionic, diamniotic
O30.04- Twin pregnancy, dichorionic, diamniotic
For triplet, quadruplet, and other multiple gestations, documentation must identify the placenta status:
Two or more monochorionic fetuses
Two or more monoamniotic fetuses
Trichorionic/triamniotic (triplet pregnancy)
Quadrachorionic/quadra-amniotic (quadruplet pregnancy)
Number of chorions and amnions both equal to the number of fetuses (other multiple gestations)
There is also an unspecified group of codes in each of the multiple gestation subcategories to indicate that the number of placentas and amniotic sacs has not been documented, as well as codes to indicate that the number of placentas and amniotic sacs cannot be determined. When the provider cannot identify how many placentas and amniotic sacs are present, this should be documented to avoid assignment of an unspecified code.
Code-Specific Documentation Requirements
The next section presents some of the ICD-10-CM codes in Chapter 15 that have special documentation requirements. The focus is on frequently reported conditions that require additional or particular pieces of clinical documentation to assign the best code(s). Though not all of the codes with special documentation requirements are discussed, this section will provide a representative sample of the type of documentation elements required for complications of pregnancy, childbirth, and the puerperium. This section is organized by topic or groups of related topics.
Antepartum Hemorrhage, Abruptio Placentae, and Placenta Previa
Antepartum hemorrhage refers to any bleeding from the vagina beginning between 20-24 weeks gestation and term. The bleeding may have a benign obstetrical cause such as benign bloody show or it may indicate an obstetrical complication such as placenta previa or abruptio placentae.
Placenta previa is an obstetrical complication in which the placenta attaches low in the uterus, partially or totally obstructing the cervix and the opening to the birth canal. Complete placenta previa covers all of the cervical opening. Partial placenta previa covers part of the cervical opening. Marginal placenta previa lies right on the border of the cervix. A low-lying placenta is within 2 cm of the cervix, but not bordering it. Placenta previa can cause severe bleeding and other complications, and necessitates a cesarean delivery.
Abruptio placentae is an obstetrical complication in which the placenta separates from the wall of the uterus prior to delivery of the fetus. The separation may be apparent with bleeding from the vagina, or concealed if it occurs behind the wall of the uterus. Abruptio placentae can be life threatening to both the mother and fetus. Risk factors for developing the condition include trauma, hypertension, coagulation disorders, smoking, multiple gestation, multiparity, and infections.
There are three code categories for the three general placental complications and causes of antepartum hemorrhage. These include O44 Placenta previa; O45 Premature separation of placenta (abruptio placentae); and O46 Antepartum hemorrhage, not elsewhere classified. The codes are all specific to the trimester. Placenta previa codes are specified as with and without hemorrhage. Premature separation of the placenta may be coded as with coagulation defect (O45.0-), other (O45.8X-), or unspecified (O45.9-) premature separation. Category O46 reports antepartum hemorrhage, not elsewhere classified, which also includes a subcategory (046.0-) for antepartum hemorrhage with coagulation defect. Combination codes are provided for separation of placenta and antepartum hemorrhage that identify the condition with the specific type of coagulation defect, such as afibrinogenemia, disseminated intravascular coagulation, and other coagulation defect.
Coding and Documentation Requirements
Identify antepartum condition:
Antepartum hemorrhage (not specified as with or due to abruptio placentae)
With coagulation defect:
»Afibrinogenemia
»Disseminated intravascular coagulation
»Other coagulation defect
»Unspecified coagulation defect
Other antepartum hemorrhage
Unspecified antepartum hemorrhage
Placenta previa
Complete
»With hemorrhage
»Without hemorrhage/NOS
Partial or Marginal
»With hemorrhage
»Without hemorrhage/NOS
Low lying
»With hemorrhage
»Without hemorrhage/NOS
Premature separation of placenta/abruptio placentae
With coagulation defect:
»Afibrinogenemia
»Disseminated intravascular coagulation
»Other coagulation defect
»Unspecified coagulation defect
Other premature separation of placenta
Unspecified premature separation of placenta
Identify trimester:
First (less than 14 weeks 0 days)
Second (14 weeks 0 days to less than 28 weeks 0 days)
Third (28 weeks 0 days until delivery)
Unspecified
ICD-10-CM Code/Documentation
O46.00-
Antepartum hemorrhage with coagulation defect, unspecified
O46.01-
Antepartum hemorrhage with afibrinogenemia
O46.02-
Antepartum hemorrhage with disseminated intravascular coagulation
O46.09-
Antepartum hemorrhage with other coagulation defect
O46.8X-
Other antepartum hemorrhage
O46.9-
Antepartum hemorrhage, unspecified
Note: Subcategory codes require a fifth or sixth character to identify the trimester.
O45.00-
Premature separation of placenta with coagulation defect, unspecified
O45.01-
Premature separation of placenta with afibrinogenemia
O45.02-
Premature separation of placenta with disseminated intravascular coagulation
O45.09-
Premature separation of placenta with other coagulation defect
O45.8X-
Other premature separation of placenta
O45.9-
Premature separation of placenta, unspecified
Note: Subcategory codes require a fifth or sixth character to identify the trimester.
O44.0-
Complete placenta previa NOS or without hemorrhage
O44.1-
Complete placenta previa with hemorrhage
O44.2-
Partial placenta previa without hemorrhage
O44.3-
Partial placenta previa with hemorrhage
O44.4-
Low lying placenta NOS or without hemorrhage
O44.5-
Low lying placenta with hemorrhage
Note: Subcategory codes require a fifth character to identify the trimester
Note: Category O46 does not include threatened abortion or other conditions classified in category O20 nor does it include hemorrhage during labor and delivery (intrapartum hemorrhage) classified in category O67.
Documentation and Coding Example
Twenty-nine-year-old Caucasian female at 34 weeks gestation with a history of placental abruption during previous pregnancy is admitted with Grade 1 placental separation. She is not in active labor. She is hemodynamically stable and fetal BPP WNL. A 16-gauge angiocath is placed in her left arm for infusion of LR. She is placed on cardiac and fetal monitors. Family allowed in to visit. Repeat US is ordered for this evening. Lab tests are ordered q 8 hours and include CBC w/platelets and smear for schistocytes, electrolytes, PT, PTT, fibrinogen, D-dimer, FDP. She has been typed and crossmatched for PRBCs, platelets, and FFP. Critical care specialist (CCS) and hematology fellow have been contacted due to increased risk factor for DIC from placental abruption.
L&D Observation Day 2: Patient has a rising DIC Score with platelet count at <50,000, D-dimer/FDP 3.0, Fibrinogen 1.0 and PT < 40 % for a total score of 6. Decision made to perform emergency C-section under general anesthesia with transfer to SICU for recovery and treatment.
Post Delivery Note: Patient tolerated surgery very well and is under care of CCS and hematology in the SICU. Infant taken to NICU but is reported to be doing well without oxygen or respiratory support.
Post Op Day 1-CCS Note: Patient is extubated, alert and oriented. She is responding to FFP and platelet transfusions and has not shown signs of progressive organ involvement from DIC. She will remain in SICU for another 24 hours and if she continues to show stable bleeding times will consider transfer to OB floor so she can be with her infant.
OB Floor Notes: Patient transferred to OB unit post-op day 2, hemodynamically stable. Remainder of hospital stay of mother and infant unremarkable.
Discharge Diagnosis: Emergency C-section for placental abruption complicated by DIC
Diagnosis Code(s)
O45.023
Premature separation of placenta with disseminated intravascular coagulation, third trimester
Z37.0
Single live birth
Coding Note(s)
A combination code is reported that identifies both the premature separation of the placenta and the specific type of coagulation defect. Antepartum hemorrhage due to coagulation defects is not reported additionally because there is an excludes note under O46 and it would only be reported if the patient had an antepartum hemorrhage due to coagulation defect without documentation of premature separation of the placenta.
Diabetes Mellitus (Pre-existing) in Pregnancy
Pre-existing diabetes mellitus is a significant complicating factor in pregnancy and requires careful management of nutrition and insulin or other diabetes medications during the pregnancy to keep blood sugar levels under control.
Subcategory codes in category O24 Diabetes mellitus in pregnancy, childbirth, and the puerperium provide information on the type of diabetes. There are specific codes for type 1, type 2, other pre-existing diabetes mellitus, unspecified pre-existing DM, and unspecified diabetes mellitus in pregnancy. Codes for pre-existing diabetes are also specific to the trimester, childbirth, or the puerperium. A secondary code is also required from category E08, E09, E10, E11, or E13 to identify further any manifestations or complications.
Coding and Documentation Requirements
Identify type of pre-existing diabetes mellitus:
Diabetes mellitus Type 1
Diabetes mellitus Type 2
Other pre-existing type of diabetes, which includes:
Diabetes due to underlying conditions (such as congenital rubella, Cushing’s syndrome, cystic fibrosis, malignant neoplasm, malnutrition, pancreatitis, and other diseases of the pancreas)
Drug or chemical induced diabetes
Other specified type, which includes:
»Diabetes mellitus due to genetic defects of beta-cell function
»Diabetes mellitus due to genetic defects of insulin action
»Postpancreatectomy diabetes mellitus
»Secondary diabetes mellitus NEC
Unspecified type diabetes mellitus
Identify trimester:
First (less than 14 weeks 0 days)
Second (14 weeks 0 days to less than 28 weeks 0 days)
Third (28 weeks 0 days until delivery)
Unspecified
Use an additional code from the appropriate category (E08, E09, E10, E11, E13) to identify further any manifestations/complications:
No complications
Ketoacidosis
With coma
Without coma
Kidney complications
Diabetic nephropathy
Diabetic chronic kidney disease
Other diabetic kidney complication
Ophthalmic complications
Diabetic retinopathy
»Mild nonproliferative
With macular edema
Without macular edema
»Moderate nonproliferative
With macular edema
Without macular edema
»Severe nonproliferative
With macular edema
Without macular edema
»Proliferative
With macular edema
Without macular edema
With traction retinal detachment involving the macula
With traction retinal detachment not involving the macula
With combined traction retinal detachment and rhegmatogenous retinal detachment
»Stable proliferative
»Unspecified
With macular edema
Without macular edema
Diabetic cataract
Diabetic macular edema, resolved following treatment
Other diabetic ophthalmic complication
Neurological complications
Diabetic amyotrophy
Diabetic autonomic (poly)neuropathy
Diabetic mononeuropathy
Diabetic polyneuropathy
Other diabetic neurological complication
Unspecified diabetic neuropathy
Circulatory complications
Diabetic peripheral angiopathy
»With gangrene
»Without gangrene
Other specified circulatory complications
Diabetic arthropathy
Neuropathic
Other
Diabetic skin complications
Diabetic dermatitis
Foot ulcer
Other skin ulcer
Other skin complications
Diabetic oral complications
Periodontal disease
Other oral complications
Hyperglycemia
Hyperosmolarity
with coma
without coma
Hypoglycemia
With coma
Without coma
Other specified complication
Unspecified complication
ICD-10-CM Code/Documentation
O24.011
Pre-existing type 1 diabetes mellitus, in pregnancy, first trimester
O24.012
Pre-existing type 1 diabetes mellitus, in pregnancy, second trimester
O24.013
Pre-existing type 1 diabetes mellitus, in pregnancy, third trimester
O24.019
Pre-existing type 1 diabetes mellitus, in pregnancy, unspecified trimester
O24.02
Pre-existing type 1 diabetes mellitus, in childbirth
O24.03
Pre-existing type 1 diabetes mellitus, in the puerperium
O24.111
Pre-existing type 2 diabetes mellitus, in pregnancy, first trimester
O24.112
Pre-existing type 2 diabetes mellitus, in pregnancy, second trimester
O24.113
Pre-existing type 2 diabetes mellitus, in pregnancy, third trimester
O24.119
Pre-existing type 2 diabetes mellitus, in pregnancy, unspecified trimester
O24.12
Pre-existing type 2 diabetes mellitus, in childbirth
O24.13
Pre-existing type 2 diabetes mellitus, in the puerperium
O24.311
Unspecified pre-existing diabetes mellitus in pregnancy, first trimester
O24.312
Unspecified pre-existing diabetes mellitus in pregnancy, second trimester
O24.313
Unspecified pre-existing diabetes mellitus in pregnancy, third trimester
O24.319
Unspecified pre-existing diabetes mellitus in pregnancy, unspecified trimester
O24.32
Unspecified pre-existing diabetes mellitus in childbirth
O24.33
Unspecified pre-existing diabetes mellitus in the puerperium
O24.811
Other pre-existing diabetes mellitus in pregnancy, first trimester
O24.812
Other pre-existing diabetes mellitus in pregnancy, second trimester
O24.813
Other pre-existing diabetes mellitus in pregnancy, third trimester
O24.819
Other pre-existing diabetes mellitus in pregnancy, unspecified trimester
O24.82
Other pre-existing diabetes mellitus in childbirth
O24.89
Other pre-existing diabetes mellitus in the puerperium
Documentation and Coding Example
Twenty-year-old Caucasian female at 35 weeks gestation presents to high risk OB clinic for scheduled visit. PMH is significant for IDDM, onset at age 7. Her diabetes was poorly controlled prior to this unplanned pregnancy and has remained poorly controlled during the pregnancy. She experienced low BGL early in pregnancy and high levels in her second and third trimesters. Hemoglobin A1C has ranged from 7.8 to 10.4. She is using NPH and Novolog insulin. She states she has not felt well for several days and home BGL has been consistently elevated for the past two days. On examination, this is a thin, ill-appearing young woman. Temperature 99, HR 120, RR 20, BP 94/50. Strong ketotic breath odor, mucous membranes dry. Breath sounds clear. CBC shows no sign of infection. Urine positive for large amount of ketones, glucose, no WBC, casts, or bacteria present. Serum glucose 480 mg/dl.
Diagnosis: Uncontrolled type 1 diabetes mellitus complicated by ketoacidosis.
Plan: Admit under care of OB service for treatment of ketoacidosis and fetal monitoring.
Diagnosis Code(s)
O24.013
Pre-existing diabetes mellitus, type 1, in pregnancy, third trimester
E10.10
Type 1 diabetes mellitus with ketoacidosis without coma
Z3A.35
35 weeks gestation of pregnancy
Coding Note(s)
Uncontrolled diabetes is not an axis of classification for coding diabetes. When the diabetes is documented as uncontrolled, the specified type of diabetes is reported with hyperglycemia, which is listed under other specified complications. In this case, the type I diabetes with ketoacidosis means that the condition is already complicated by hyperglycemia, hyperketonemia, and metabolic acidosis. For this reason, code E10.65 Type 1 diabetes mellitus with hyperglycemia has not been reported in addition to E10.10.
Ectopic Pregnancy
An ectopic pregnancy occurs when a fertilized egg implants outside the uterus. The most common site of ectopic pregnancy is in one of the fallopian tubes, but implantation can also occur within an ovary, the abdominal cavity, or in the uterine cervix.
Ectopic pregnancy is classified in category O00. Codes are classified by site, laterality for tubal and ovarian sites, and whether or not there is a viable intrauterine pregnancy. There is an Excludes1 note for continuing pregnancy in multiple gestation after abortion of one fetus or more which is reported with codes in subcategories O31.1- or O31.3-. Ectopic pregnancies also include those that have ruptured.
Documentation and Coding Requirements
Identify site of ectopic pregnancy:
Abdominal
Ovarian
Tubal
Other specified site
Unspecified ectopic pregnancy
Identify whether or not a viable intrauterine pregnancy is also present:
With intrauterine pregnancy
Without intrauterine pregnancy
Identify laterality for fallopian tube and ovarian ectopic sites:
Left
Right
Unspecified
Use additional code from category O08 to identify any associated complications.
ICD-10-CM Code/Documentation
O00.00
Abdominal pregnancy without intrauterine pregnancy
O00.01
Abdominal pregnancy with intrauterine pregnancy
O00.101
Right tubal pregnancy without intrauterine pregnancy
O00.102
Left tubal pregnancy without intrauterine pregnancy
O00.109
Unspecified tubal pregnancy without intrauterine pregnancy
O00.111
Right tubal pregnancy with intrauterine pregnancy
O00.112
Left tubal pregnancy with intrauterine pregnancy
O00.119
Unspecified tubal pregnancy with intrauterine pregnancy
O00.201
Right ovarian pregnancy without intrauterine pregnancy
O00.202
Left ovarian pregnancy without intrauterine pregnancy
O00.209
Unspecified ovarian pregnancy without intrauterine pregnancy
O00.211
Right ovarian pregnancy with intrauterine pregnancy
O00.212
Left ovarian pregnancy with intrauterine pregnancy
O00.219
Unspecified ovarian pregnancy with intrauterine pregnancy
O00.80
Other ectopic pregnancy without intrauterine pregnancy
O00.81
Other ectopic pregnancy with intrauterine pregnancy
O00.90
Unspecified ectopic pregnancy without intrauterine pregnancy
O00.91
Unspecified ectopic pregnancy with intrauterine pregnancy
Documentation and Coding Example
Thirty-three-year-old Hispanic female presents to OB/GYN with vaginal spotting and right lower abdominal pain. Patient initially called one week ago after a home pregnancy test was positive and was given an appointment 2 weeks out but she called back today and spoke to the triage nurse when she developed spotting and pain over the past 2 days. She is being seen emergently by the nurse-midwife. The patient is a G1 P1, who had an uncomplicated pregnancy with vaginal delivery at term 4 years ago. Her daughter is doing well. She states this is a planned pregnancy and she and her husband are excited. They have not yet told their daughter or other family members because she has just felt things were “not quite right” with the pregnancy. She admits to fatigue and increased urination but no nausea, vomiting, or food aversions which were early symptoms with her first pregnancy. T 98.8, P 76, R 12, BP 102/66, Ht. 66 inches, Wt. 133 lbs. On examination, this is a well-developed, well nourished, anxious young woman who looks her stated age. Color is pink, skin warm and dry. Heart rate regular without murmur, rubs, bruits. Extremities without evidence of clubbing. Breath sounds clear and equal bilaterally. Spine with normal curvature, no CVA tenderness is appreciated. Abdomen flat, bowel sounds present. Liver palpated at RCM, spleen is not palpated. No tenderness in upper quadrants but moderate tenderness in lower quadrants. External genitalia normal in appearance, no vaginal discharge noted. Speculum is easily inserted but there is pain when it is opened. Vaginal mucosa is dark red, cervix is closed with faint bluish discoloration consistent with pregnancy. Speculum was withdrawn and bimanual exam reveals CMT and fullness in the right adnexa. Left adnexa feels normal in size as does the uterus. Legs were brought out of stirrups and she remains resting on the examination table awaiting a vaginal US by the physician. Blood is drawn for quantitative beta hCG, CBC, PT, PTT and a comprehensive metabolic panel. They are sent to lab with request for stat analysis.
Attending OB/GYN note: Vaginal exam is consistent with the findings by midwife. Transvaginal US reveals a normal size uterus without gestational sac or embryo present. The left ovary and fallopian tube are normal size and shape. The right ovary is difficult to visualize due to swelling of the distal end of the right fallopian tube. A mass appears to be present in the tube but no bleeding or excess fluid is appreciated in the cul de sac or surrounding the right adnexa. Labs are reviewed and beta hCG level is 3850 mlU/mL, Hct. 38. Renal and hepatic functions, bleeding times WNL.
Impression: Nonviable, right ectopic tubal pregnancy. Patient and her husband are apprised of findings and offered condolences. Medical and surgical treatment plans are explained and discussed. Patient is an excellent candidate for methotrexate.
Plan: Administer Methotrexate 60 mg IM today. Patient to return to clinic in 4 days for repeat beta hCG and examination. She is advised to stop taking all vitamins or supplements that contain folic acid and she may take acetaminophen for pain. She should expect a decrease in abdominal pain and increase in vaginal spotting or bleeding over the next few days. If she experiences dizziness, fever, or pain that is not controlled by acetaminophen she should call immediately or go directly to the ED.
Diagnosis Code(s)
O00.101
Right tubal pregnancy without intrauterine pregnancy
Coding Note(s)
Tubal pregnancy that is not otherwise specified defaults to O00.10- for tubal pregnancy without intrauterine pregnancy. In this case, the physician has already documented a normal uterus without a gestational sac or embryo present.
Gestational Diabetes/Abnormal Glucose Tolerance in Pregnancy
Gestational diabetes is glucose intolerance during pregnancy. Hormones produced by the placenta block insulin receptors in the mother causing maternal blood glucose levels to rise. Gestational diabetes is diagnosed using glucose tolerance testing. Generally, an initial glucose tolerance test is performed. If the initial test is abnormal, usually considered to be a blood sugar level above 140 mg/dL or 7.8 mmol/L (although definitions of elevated blood sugar levels may vary by lab), a second glucose tolerance test is performed to make a diagnosis of gestational diabetes. It is possible to have an abnormal glucose tolerance test without a diagnosis of gestational diabetes.
Abnormal glucose tolerance test (O99.81-) is not synonymous with gestational diabetes. In addition, gestational diabetes reported with codes in subcategory O24.4 requires additional documentation related to whether the condition is being controlled by diet, oral hypoglycemics, or whether the patient requires insulin to control the condition.
Coding and Documentation Requirements
Identify the condition:
Abnormal glucose complicating pregnancy, childbirth, and puerperium (O99.81-)
Gestational diabetes (O24.4-)
Identify the maternal episode of care:
Pregnancy
Childbirth
Puerperium
For gestational diabetes, specify method of control:
Diet controlled
Controlled by oral hypoglycemic drugs
Insulin controlled
Unspecified control
ICD-10-CM Code/Documentation
O99.810
Abnormal glucose complicating pregnancy
O99.814
Abnormal glucose complicating childbirth
O99.815
Abnormal glucose complicating the puerperium
O24.410
Gestational diabetes mellitus in pregnancy, diet-controlled
O24.414
Gestational diabetes mellitus in pregnancy, insulin controlled
O24.415
Gestational diabetes mellitus in pregnancy, controlled by oral hypoglycemic drugs
O24.419
Gestational diabetes mellitus in pregnancy, unspecified control
O24.420
Gestational diabetes mellitus in childbirth, diet-controlled
O24.424
Gestational diabetes mellitus in childbirth, insulin controlled
O24.425
Gestational diabetes mellitus in childbirth, controlled by oral hypoglycemic drugs
O24.429
Gestational diabetes mellitus in childbirth, unspecified control
O24.430
Gestational diabetes mellitus in the puerperium, diet-controlled
O24.434
Gestational diabetes mellitus in the puerperium, insulin controlled
O24.435
Gestational diabetes mellitus in the puerperium, controlled by oral hypoglycemic drugs
O24.439
Gestational diabetes mellitus in the puerperium, unspecified control
Note: There are no codes for gestational diabetes or abnormal glucose for an unspecified episode of care. There must be clear documentation related to whether the condition is affecting the pregnancy, childbirth, or the puerperium.
Documentation and Coding Example
Patient is a 26-year-old G3 P2 Hispanic female at 33 weeks gestation who presents to high risk OB clinic for follow-up appointment for gestational diabetes. Patient had an abnormal glucose tolerance test 6 weeks ago and has been followed by the dietician for nutritional counseling and home blood glucose monitoring. She struggled with compliance because she primarily eats rice and beans and was started on insulin therapy 2 weeks ago. Fasting BGL and postprandial levels have been WNL since beginning insulin therapy. On examination, this is a petite, young woman who is accompanied by her 5-year-old daughter and 3-year-old son. Wt.125 which is unchanged from last visit. HR 88, RR 14, BP 138/86. Gravid abdomen, single fetus in breech position. FHR 150. No edema noted in extremities. NST unremarkable, biophysical profile shows normal for GA fetus, adequate amniotic fluid. Patient will continue on insulin therapy.
Diagnosis Code(s)
O24.414
Gestational diabetes mellitus in pregnancy, insulin controlled
Z3A.33
33 weeks gestation of pregnancy
Coding Note(s)
The combination code identifies both the gestational diabetes mellitus in pregnancy and the insulin use, so only a single code is required.
Hydatidiform Mole
In molar pregnancy, also referred to as a hydatidiform mole or gestational trophoblastic disease, a non-viable fertilized egg implants in the uterus and forms a mass of chorionic villi tissue, as minute, elongated projections grow out from the chorion, the protective, nutritive membrane that attaches the new embryo growth to the uterus. A complete mole contains no fetal tissue and combines one duplicated or two unique sperm cells with an egg cell that does not contain maternal DNA, creating a 46XX or 46XY cell with only paternal DNA. This type of mole has no embryonic tissue but does have chorionic tissue, which increases the risk for a type of malignancy of the trophoblastic cells called choriocarcinoma. A partial mole contains some fetal tissue and combines an egg cell containing maternal DNA with one duplicated or two unique sperm cells yielding a 69XXX or 69XXY chromosomal complement cell (or in some cases, tetraploidy with more than 69 chromosomes) having both maternal and paternal DNA. This type of mole may contain both embryonic and chorionic tissue.
Molar pregnancy (category O01) is differentiated as classical or complete hydatidiform mole, incomplete or partial hydatidiform mole, and unspecified hydatidiform mole. There is an excludes note for chorioadenoma (destruens) and for hydatidiform mole documented as malignant, which are both reported with code D39.2 Neoplasm of uncertain behavior of placenta.
Coding and Documentation Requirements
Identify type of hydatidiform mole:
Classical (complete)
Incomplete and partial
Unspecified, which includes:
Trophoblastic disease not otherwise specified
Vesicular mole not otherwise specified
ICD-10-CM Code/Documentation
O01.0
Classical hydatidiform mole
O01.1
Incomplete and partial hydatidiform mole
O01.9
Hydatidiform mole unspecified
Documentation and Coding Example
Patient is a thirty-two-year-old Caucasian female referred to OB-GYN by her midwife after it was determined on US that she had a non-viable pregnancy. The patient and her husband have a 2 ½-year-old son and were actively trying to conceive a second child. Patient is a yoga instructor and strict vegetarian. First pregnancy uncomplicated although she gained only 15 lbs., with a midwife assisted home birth of a healthy 6 lb. infant at term. Patient states this pregnancy has felt different from the start, with severe nausea and vomiting resulting in a weight loss of 5 lbs. She feels weak and exhausted, notices that her hands shake all the time and her heart often feels like it is racing. Temperature 99, HR 98, RR 14, BP 138/80, Wt. 103 lbs. Ht. 64 inches. On examination, this is a well-groomed, thin, anxious appearing woman who looks younger than her stated age. Neck is supple, thyroid can be palpated and feels normal in size. Skin warm, dry to touch. HR regular, without gallop or rub. Breath sounds clear, equal bilaterally. Abdomen soft with active bowel sounds. Uterus can be palpated above the pubis, size consistent with a 12 to 14-week gestation, patient’s dates indicate a pregnancy at only 7-8 weeks. US shows a complex uterine mass with cystic structure but no fetus or yolk sac. Two theca lutein cysts are present on the right ovary approximately 6 cm and 8 cm in size. Patient and her husband are advised that the US findings are quite consistent with classical molar pregnancy and a D & C is the recommended treatment. Patient and husband agree, OR is available tomorrow afternoon. Patient sent to OPS for pre-op blood work, chest x-ray, and anesthesia consult.
Pre Op Note: CBC shows mild anemia. PT, PTT, comprehensive metabolic panel all WNL. HCG and thyroxin levels are elevated. Chest x-ray negative.
Post Op Note: Abnormal tissue successfully evacuated with D & C under general anesthesia. Minimal bleeding. Tissue to pathology for identification. Patient discharged home in good condition with f/u appointment in 1 week. Pelvic rest until that time.
Office Visit Note: Patient is seen with her husband. She is recovering well. Nausea, vomiting, restlessness, tremors, and racing heart have all abated. She reports vaginal spotting x 2 days following procedure, now resolved. They are advised that pathology report showed an androgenetic homozygous complete molar pregnancy. She is scheduled for weekly blood draw to monitor HCG levels, advised to continue pelvic rest x 3 more weeks and return at that time for recheck and to discuss contraceptive methods.
Diagnosis Code(s)
O01.0
Classical hydatidiform mole
Coding Note(s)
Any complications following ectopic molar pregnancy such as genital tract and pelvic infection, delayed or excessive hemorrhage, embolism, shock, renal failure, metabolic disorders, damage to pelvic organs, other venous complications, cardiac arrest, sepsis, or urinary tract infection are reported separately with codes from category O08.
Hypertension Complicating Pregnancy, Childbirth and the Puerperium
Hypertension complicating pregnancy is classified based on whether the hypertension is pre-existing or pregnancy-induced. There are 6 categories for these conditions and additional subcategories within each category. All codes require documentation of the trimester. In order to describe the documentation requirements for hypertension complicating pregnancy, it is necessary to subdivide this section into codes covering pre-existing hypertension and codes covering gestational hypertension, pre-eclampsia, and eclampsia.
Pre-Existing Hypertension Complicating Pregnancy, Childbirth, Puerperium
Pre-existing hypertension in pregnancy may be either primary or secondary. Primary hypertension is sometimes referred to as essential hypertension, which is abnormally elevated blood pressure levels that have no particular underlying cause. Secondary hypertension refers to abnormally elevated blood pressure levels due to an underlying condition or cause. Preexisting hypertension can be complicated by pre-eclampsia or eclampsia. Pre-eclampsia is hypertension (BP >140/90 mmHg) after the 20th week of gestation (and up to 6 weeks postpartum), with proteinuria. Pre-eclampsia may be considered severe with a BP >160/110 and additional symptoms such as edema and epigastric pain. Eclampsia is a life-threatening condition characterized by hypertension (BP >140/90 mmHg) after the 20th week of gestation (and up to 6 weeks postpartum), with proteinuria, and tonic-clonic (motor) seizures. Seizures are often preceded by headache, nausea, vomiting, and/or cortical blindness.
Hypertension is not differentiated as benign or malignant. It is classified in category O10 as pre-existing essential hypertension or pre-existing hypertensive heart and/or chronic kidney disease. There is also a category for pre-existing hypertension with preeclampsia (011). Hypertension complicated by pre-eclampsia requires an additional code from category O10 to identify the type of hypertension. An additional code is required from categories I11-I15 to identify the specified type of hypertensive heart and/or chronic kidney disease, or the specific type of secondary hypertension. No additional code is required for patients who have pre-existing essential hypertension.
Coding and Documentation Requirements
Identify pre-existing hypertension complicating pregnancy, childbirth, or the puerperium:
Essential hypertension
Hypertensive chronic kidney disease
Hypertensive heart disease
Hypertensive heart and chronic kidney disease
Secondary hypertension
Unspecified pre-existing hypertension
Identify complication as occurring during pregnancy, childbirth, puerperium:
Pregnancy
First trimester (less than 14 weeks 0 days)
Second trimester (14 weeks 0 days to less than 28 weeks 0 days)
Third trimester (28 weeks 0 days until delivery)
Unspecified trimester
Childbirth
Puerperium
For pre-existing hypertension documented as hypertensive heart and/or hypertensive chronic kidney disease or secondary hypertension, assign an additional code from categories I11-I13 or I15 to identify the type of hypertension more specifically.
ICD-10-CM Code/Documentation
O10.11-
Pre-existing hypertensive heart disease complicating pregnancy
O10.12
Pre-existing hypertensive heart disease complicating childbirth
O10.13
Pre-existing hypertensive heart disease complicating the puerperium
O10.01-
Pre-existing essential hypertension complicating pregnancy
O10.02
Pre-existing essential hypertension complicating childbirth
O10.03
Pre-existing essential hypertension complicating the puerperium
O10.21-
Pre-existing hypertensive chronic kidney disease complicating pregnancy
O10.22
Pre-existing hypertensive chronic kidney disease complicating childbirth
O10.23
Pre-existing hypertensive chronic kidney disease complicating the puerperium
O10.41-
Pre-existing secondary hypertension complicating pregnancy
O10.42
Pre-existing secondary hypertension complicating childbirth
O10.43
Pre-existing secondary hypertension complicating the puerperium
O10.31-
Pre-existing hypertensive heart and chronic kidney disease complicating pregnancy
O10.32
Pre-existing hypertensive heart and chronic kidney disease complicating childbirth
O10.33
Pre-existing hypertensive heart and chronic kidney disease complicating the puerperium
O11.1
Pre-existing hypertension with pre-eclampsia, first trimester
O11.2
Pre-existing hypertension with pre-eclampsia, second trimester
O11.3
Pre-existing hypertension with pre-eclampsia, third trimester
O11.9
Pre-existing hypertension with pre-eclampsia, unspecified trimester
Note: Codes from category O11 need a code from category O10 assigned in addition to identify the type of hypertension
Documentation and Coding Example
Antepartum Admission Note: Thirty-three-year-old Black female admitted to antepartum observation floor with symptoms of preeclampsia at 30 weeks gestation. Patient has a 3-year history of chronic hypertension well controlled with Tenormin. She was switched to Aldomet when her pregnancy was identified and has been followed by both perinatology and cardiology for high risk pregnancy. Temperature 99, HR 72, RR 12, BP 152/100. Patient is a G1 P0 and states she and her husband were surprised when she became pregnant, having tried to conceive unsuccessfully for 10 years. They are thrilled to be having a baby and know that it is a boy. There is no evidence of IUGR on US and biophysical profiles have been WNL. Decision was made today at routine OB appointment to admit for monitoring of elevated BP, worsening edema, and protein in her urine. IV started in left arm and blood drawn for CBC, coagulation studies, comprehensive metabolic panel, type and hold. Magnesium sulfate infusion started. IM betamethasone also administered. Patient on bedrest, right side as much as tolerated.
Diagnosis: Pregnancy at 30 weeks with pre-existing hypertension and superimposed pre-eclampsia
Diagnosis Code(s)
O11.3
Pre-existing hypertension with pre-eclampsia, third trimester
O10.013
Pre-existing essential hypertension complicating pregnancy, third trimester
Z3A.30
30 weeks gestation of pregnancy
Coding Note(s)
There is no documentation of hypertensive heart or chronic kidney disease so the code for essential hypertension complicating pregnancy is reported as the additional code to identify the type of hypertension. When the weeks of gestation are documented, a code from category Z3A should be assigned.
Gestational Hypertension/Pre-Eclampsia/Eclampsia
Gestational hypertension, also called pregnancy-induced hypertension, is an arterial blood pressure >140/90 mmHg, without proteinuria (protein present in urine) in a pregnant woman without previously documented hypertension after 20 weeks gestation. Pre-eclampsia is hypertension (BP >140/90 mmHg) after the 20th week of gestation, and up to 6 weeks postpartum, with proteinuria. Pre-eclampsia may be considered severe with a BP >160/110 and additional symptoms such as edema and epigastric pain. In pre-eclampsia, the hypertension may be pre-existing or pregnancy-induced. Eclampsia is a life-threatening condition characterized by hypertension with a BP >140/90 mmHg after the 20th week of gestation, and up to 6 weeks postpartum, with proteinuria, and tonic-clonic (motor) seizures. Seizures are often preceded by headache, nausea, vomiting, and/or cortical blindness. In eclampsia, the hypertension may be preexisting or pregnancy-induced.
Gestational hypertension is clarified as meaning pregnancy-induced hypertension. The category description for O13 also specifies that the gestational hypertension is without significant proteinuria, although if the diagnostic statement does not state that the condition is complicated by significant proteinuria, then this category is the default for gestational hypertension NOS. If the patient has gestational hypertension with significant proteinuria, the condition is classified as either mild to moderate pre-eclampsia (O14.0-), severe pre-eclampsia (O14.1-), or severe pre-eclampsia with hemolysis, elevated liver enzymes, and low platelet count (HELLP syndrome) (O14.2-). There is also a code for unspecified pre-eclampsia (O14.9). Eclampsia is reported with codes from category O15.
Coding and Documentation Requirements
Identify gestational hypertension, pre-eclampsia, or eclampsia:
Gestational hypertension without significant proteinuria
Pre-eclampsia
Mild to moderate
Severe
HELLP syndrome
Unspecified
Eclampsia
Unspecified maternal hypertension
Identify stage of pregnancy in which complication occurs:
Pregnancy
Childbirth/labor
Puerperium
Identify trimester of pregnancy:
First (less than 14 weeks 0 days)
Second (14 weeks 0 days to less than 28 weeks 0 days)
Third (28 weeks 0 days until delivery)
Unspecified
ICD-10-CM Code/Documentation
O13.1
Gestational [pregnancy-induced] hypertension without significant proteinuria, first trimester
O13.2
Gestational [pregnancy-induced] hypertension without significant proteinuria, second trimester
O13.3
Gestational [pregnancy-induced] hypertension without significant proteinuria, third trimester
O13.9
Gestational [pregnancy-induced] hypertension without significant proteinuria, unspecified trimester
O14.00
Mild to moderate pre-eclampsia, unspecified trimester
O14.02
Mild to moderate pre-eclampsia, second trimester
O14.03
Mild to moderate pre-eclampsia, third trimester
O14.04
Mild to moderate pre-eclampsia, complicating childbirth
O14.05
Mild to moderate pre-eclampsia, complicating the puerperium
O14.10
Severe pre-eclampsia, unspecified trimester
O14.12
Severe pre-eclampsia, second trimester
O14.13
Severe pre-eclampsia, third trimester
O14.14
Severe pre-eclampsia, complicating childbirth
O14.15
Severe pre-eclampsia, complicating the puerperium
O14.20
HELLP syndrome, unspecified trimester
O14.22
HELLP syndrome, second trimester
O14.23
HELLP syndrome, third trimester
O14.24
HELLP syndrome, complicating childbirth
O14.25
HELLP syndrome, complicating the puerperium
O15.00
Eclampsia complicating pregnancy, unspecified trimester
O15.02
Eclampsia complicating pregnancy, second trimester
O15.03
Eclampsia complicating pregnancy, third trimester
O15.1
Eclampsia complicating labor
O15.2
Eclampsia complicating the puerperium
Note: Codes are only available for pre-eclampsia and eclampsia in the second or third trimester in a patient without pre-existing hypertension. This is because these conditions do not occur in the first trimester.
Documentation and Coding Example
Twenty-seven-year-old Black female is seen in OB Clinic at 34 weeks gestation. Patient is a G1P1 with a single gestation pregnancy that is now complicated by gestational hypertension. She is accompanied by her husband. Temperature 99, HR 70, RR 12, BP 150/96. Normal BP and urine protein until 4 weeks ago when she presented with BP 142/90 and 1+ protein on dipstick. Fetal ultrasound at that time showed normal fluid levels which have remained normal on subsequent US. BP has been monitored weekly by CNM/OB along with urine protein checks. On examination: PERRL, fundoscopic exam normal. Neck supple without lymphadenopathy, thyroid normal size/shape. She c/o mild nasal congestion which has been present throughout pregnancy. No history of allergies or respiratory problems. Breath sounds clear, equal bilaterally. HR regular without gallop or rub. Gravid abdomen, vertex presenting, not engaged. FHR 144. No hepatic tenderness. Reflexes are brisk, mild dependent edema noted in feet and ankles. Dipstick urine is 2+ protein and sample sent to lab for UA with Protein/Creatinine ratio. Blood drawn for CBC, comprehensive metabolic panel, coagulation studies. US reveals single female fetus with normal amniotic fluid levels. NST shows good variability with fetal activity. RTC in 1 week for BP check, US, and NST. Impression: Moderate pre-eclampsia. Patient is instructed on symptoms that need immediate attention and how to reach on call staff after clinic hours. She is advised that she will be seen in clinic 2 x week for monitoring and that she may be delivered early. RTC in 4 days.
Diagnosis Code(s)
O14.03
Mild to moderate pre-eclampsia, third trimester
Z3A.34
34 weeks gestation of pregnancy
Coding Note(s)
Gestational hypertension with significant proteinuria is coded as pre-eclampsia. The patient has both proteinuria and mild dependent edema, but the pre-eclampsia is specified as moderate not severe and so moderate pre-eclampsia is reported. Severe pre-eclampsia requires documentation of proteinuria and/or edema specified as severe.
Infection of Amniotic Sac/Membranes
An inflammation or infection of the amniotic cavity may involve the amniotic sac, which consists of a pair of membranes, the chorion and the amnion, known as the fetal membranes that exist during pregnancy between the developing fetus and the mother. When the pair of membranes that compose the amniotic sac is affected, the condition is referred to as chorioamnionitis, an inflammation of the chorion and the amnion. The chorion is the outermost membrane on the maternal side surrounding the embryo and contributing to the placenta’s formation, and the amnion is the innermost membrane on the fetal side. Chorioamnionitis is usually caused by a bacterial infection. Bacteria can be introduced from the urogenital tract via vaginal examinations performed in late pregnancy and/or during labor. Symptoms may include maternal fever, uterine tenderness, and foul smelling vaginal discharge. Mild chorioamnionitis is characterized by infiltration of neutrophils in the chorionic plate. Moderate infection may involve necrosis and/or abscess of the subamniotic tissue or fetal membrane. With severe involvement, there may be vasculitis of the umbilical vessels and/or inflammation of the umbilical cord and connective tissue (funisitis).
An inflammation or infection of the amniotic cavity may also involve the placenta. Placentitis is an inflammation of the placenta, an organ that secretes hormones and provides nutrition, waste elimination, and gas exchange to the developing fetus. Placentitis may cause miscarriage, preterm labor, and/or placental retention post-delivery. TORCH infections are the most common cause of placentitis and include toxoplasmosis (T), other infections (O- coxsackievirus, varicella-Zoster virus, HIV, Parvovirus B19), Rubella (R), cytomegalovirus (C) and herpes simplex-2 (H).
There are specific codes for chorioamnionitis (O41.12-) and placentitis (O41.14-). Amnionitis and membranitis are reported with the code for chorioamnionitis.
Coding and Documentation Requirements
Identify site of infection of the amniotic sac and membranes:
Chorioamnionitis
Placentitis
Unspecified site
Identify trimester:
First (less than 14 weeks 0 days)
Second (14 weeks 0 days to less than 28 weeks 0 days)
Third (28 weeks 0 days until delivery)
Unspecified
Identify fetus affected by complication:
Fetus 1
Fetus 2
Fetus 3
Fetus 4
Fetus 5
Other fetus
Unspecified fetus/not applicable
ICD-10-CM Code/Documentation
O41.101-
Infection of amniotic sac and membranes, unspecified, first trimester
O41.102-
Infection of amniotic sac and membranes, unspecified, second trimester
O41.103-
Infection of amniotic sac and membranes, unspecified, third trimester
O41.109-
Infection of amniotic sac and membranes, unspecified, unspecified trimester
O41.121-
Chorioamnionitis, first trimester
O41.122-
Chorioamnionitis, second trimester
O41.123-
Chorioamnionitis, third trimester
O41.129-
Chorioamnionitis, unspecified trimester
O41.141-
Placentitis, first trimester
O41.142-
Placentitis, second trimester
O41.143-
Placentitis, third trimester
O41.149-
Placentitis, unspecified trimester
Assign 7th character for fetus affected by infection of amniotic cavity:
1 for fetus 1
2 for fetus 2
3 for fetus 3
4 for fetus 4
5 for fetus 5
9 for other fetus
0 for unspecified fetus/not applicable
Documentation and Coding Example
Thirty-six-year-old female with twin dichorionic, diamniotic gestation at 34 weeks admitted to L&D with possible PROM. Patient is a G2 P0 who suffered a 2nd trimester loss with her first pregnancy 2 years ago due to incompetent cervix. She had a cerclage placed at 12 weeks and has been on bed rest for most of this pregnancy. Temperature 100.6, HR 88, RR 16 BP 120/70. On examination, this is an anxious woman, clean and well groomed. She states that she noticed some clear fluid on her underclothes yesterday but felt it was probably urine. This morning she awoke and felt a small gush of fluid when she got out of bed to use the BR. She called her doctor who ordered her to the hospital. She denies abdominal pain or contractions, urinary frequency or burning. Vaginal exam with sterile speculum reveals fluid leaking from cervical os which is sutured closed. Fluid sample sent to lab for leukocyte count, Gram stain, pH, glucose concentration, endotoxins, lactoferrin, and cytokines. US shows male fetus (Fetus 1) in lower uterus, decreased amniotic fluid volume with particulate matter in the fluid. Placenta appears to be on the posterior wall of the uterus and cannot be easily evaluated. Female fetus (Fetus 2) is in upper uterus with adequate amniotic fluid and a normal appearing placenta located on the right side of the uterus. BPP of each fetus is WNL. IV access established with 16-gauge angiocath in patients left forearm. LR infusing at 125 cc hour for hydration. Blood drawn for CBC, Chem panel, coagulation studies, Type and hold, CRP, Alpha 1-proteinase inhibitor, serum ferritin. She had a course of betamethasone at 28 weeks so it is presumed fetal lungs are mature.
Impression: Premature rupture of membranes. Chorioamnionitis, male fetus
Plan: Admit for monitoring and administration of antibiotics
Diagnosis Code(s)
O42.913
Preterm premature rupture of membranes, unspecified as to length of time between rupture and onset of labor, third trimester
O41.1231
Chorioamnionitis, third trimester, fetus 1
O30.043
Twin pregnancy, dichorionic/diamniotic, third trimester
O34.33
Maternal care for incompetent cervix, third trimester
Z3A.34
34 weeks gestation of pregnancy
Coding Note(s)
Chorioamnionitis requires a 7th character extension to identify the fetus. In this scenario of a twin pregnancy, only the male fetus identified as fetus 1 is affected by the infection, so 7th character extension 1 is reported. Since the female fetus is not affected by the infection, no code is reported for fetus 2.
Infections of Genitourinary Tract in Pregnancy
Codes in category O23 Infections of the genitourinary tract in pregnancy, are specific to the antepartum period and the trimester must be specified. Codes from category O23 should not be used to report infections with a predominantly sexual mode of transmission, such as syphilis or gonorrhea. An additional code should be reported from category B95 or B96 to identify the organism. Codes are also specific to site. For genitourinary tract infections following delivery, a code from subcategory O86.1 Other infection of genital tract following delivery, or O86.2 Urinary tract infection following delivery, is reported. These codes are also site specific.
Coding and Documentation Requirements
Identify site for genitourinary tract infection during pregnancy (O23):
Urinary tract
Kidney
Bladder
Urethra
Other parts of urinary tract
Unspecified infection of urinary tract
Genital tract
Cervix
Ovary/tube (Salpingitis/Oophoritis)
Other part of genital tract
Unspecified site of genitourinary tract
Identify trimester:
First (less than 14 weeks 0 days)
Second (14 weeks 0 days to less than 28 weeks 0 days)
Third (28 weeks 0 days until delivery)
Unspecified
Use additional code to identify organism (B95.-, B96.-).
Identify condition/site for genitourinary tract infection following delivery (O86.1-, O86.2-):
Genital tract
Cervicitis/cervix
Endometritis/endometrium
Vaginitis/vagina
Other infection of genital tract
Urinary tract
Bladder
Kidney
Other specified site
Unspecified site
Use additional code to identify organism (B95.-, B96.-).
ICD-10-CM Code/Documentation
O23.0-
Infections of kidney in pregnancy
O23.1-
Infections of bladder in pregnancy
O23.2-
Infections of urethra in pregnancy
O23.3-
Infections of other parts of urinary tract in pregnancy
O23.4-
Unspecified infection of urinary tract in pregnancy
O23.51-
Infection of cervix in pregnancy
O23.52-
Salpingo-oophoritis in pregnancy
O23.59-
Infection of other part of genital tract in pregnancy
O23.9-
Unspecified genitourinary tract infection in pregnancy
Note: Subcategory codes require fifth or sixth characters to identify the trimester.
O86.11
Cervicitis following delivery
O86.12
Endometritis following delivery
O86.13
Vaginitis following delivery
O86.19
Other infection of genital tract following delivery
O86.20
Urinary tract infection following delivery, unspecified
O86.21
Infection of kidney following delivery
O86.22
Infection of bladder following delivery
O86.29
Other urinary tract infection following delivery
Documentation and Coding Example
Patient is a 25-year-old Native American female referred to ED by her OB with c/o crampy lower abdominal pain x 2 days which has progressed to fever and back pain. She is a G3 P2 at 16 weeks gestation and has a history of UTI with previous pregnancies. Temperature 100.4, HR 88, RR 16, BP 110/66. On examination, this is a thin, tired appearing woman who looks older than her stated age. Neck supple without lymphadenopathy. Throat, TMs clear. Mucous membranes pink, somewhat dry. HR regular without murmur or rubs. Breath sounds clear, equal bilaterally. Bilateral CVA tenderness. Abdomen soft with active BS. Spleen not palpated, liver palpated 1 cm below RCM. C/O suprapubic pain with palpation. FHR 156 by Doppler. Pelvic exam reveals red, mildly irritated external genitalia and vaginal vault. Cervix closed, no discharge noted. Urine specimen obtained by straight cath. Urine sent to lab for UA, C & S. Blood drawn for CBC, comprehensive metabolic panel, blood cultures. IV started in right arm with D5LR infusing. UA reveals WBC, casts, and large number of gram + bacteria. Patient started on IV antibiotics and admitted to medical floor.
Diagnosis: Second trimester pregnancy complicated by acute pyelonephritis.
Floor Note Day 2: Urine culture positive for Staphylococcus saprophyticus sensitive to Ciprofloxacin and Macrobid. Patient is afebrile, continues to have CVA tenderness more pronounced on right side and suprapubic discomfort. She will remain hospitalized for IV antibiotics. Urology consult requested due to history of previous UTIs during pregnancy to R/O structural abnormalities.
Urology Note: Patient seen for recurrent UTIs complicating a second trimester pregnancy. Renal US obtained, shows no abnormalities. Fever, CVA tenderness, suprapubic discomfort are resolving on antibiotics. Recommendation: Complete 7 days of IV Cipro and discharge patient on prophylactic Macrobid daily for the remainder of pregnancy.
Diagnosis Code(s)
O23.02
Infections of the kidney in pregnancy, second trimester
B95.7
Other staphylococcus as the cause of diseases classified elsewhere
Z3A.16
16 weeks gestation of pregnancy
Coding Note(s)
There is no code listed for Pregnancy, complicated by pyelonephritis. The subterm pyelitis is used to locate code O23.0-. Under O23.0 Infections of kidney in pregnancy in the tabular section is listed the alternate inclusion term ‘Pyelonephritis in pregnancy’. Three codes are required, one for the infection of the kidney in pregnancy, one to identify the infectious organism, and a third to capture the weeks of gestation.
Multiple Gestation
Multiple gestation refers to a pregnancy with more than one fetus. A twin pregnancy has two fetuses that may share a single placenta and amniotic sac, share a single placenta but have two amniotic sacs or each have a separate placenta and amniotic sac. A triplet pregnancy has three fetuses, two or more may share a placenta and/or amniotic sac. A quadruplet pregnancy has four fetuses, two or more may share a placenta and/or amniotic sac.
For multiple gestation pregnancies (O30), the number of fetuses is required along with documentation related to the number of placentas and the number of amniotic sacs. The code for conjoined twins is also listed in category O30.
Coding and Documentation Requirements
Identify multiple gestation and number of placenta/amniotic sacs as:
Twin pregnancy
Conjoined twin pregnancy
One placenta/one amniotic sac (monochorionic/monoamniotic)
One placenta/two amniotic sacs (monochorionic/diamniotic)
Two placentae/two amniotic sacs (dichorionic/diamniotic)
Unable to determine number of placenta/amniotic sacs
Unspecified number of placenta/amniotic sacs
Triplet pregnancy
Two or more monochorionic fetuses
Two or more monoamniotic fetuses
Trichorionic/triamniotic
Unable to determine number of placenta/amniotic sacs
Unspecified number of placenta/amniotic sacs
Quadruplet pregnancy
Two or more monochorionic fetuses
Two or more monoamniotic fetuses
Quadrachorionic/quadra-amniotic
Unable to determine number of placenta/amniotic sacs
Unspecified number of placenta/amniotic sacs
Other specified multiple gestation
Two or more monochorionic fetuses
Two or more monoamniotic fetuses
Number of chorions and amnions equal to the number of fetuses
Unable to determine number of placenta/amniotic sacs
Unspecified number of placenta/amniotic sacs
Unspecified multiple gestation
Identify trimester:
First (less than 14 weeks 0 days)
Second (14 weeks 0 days to less than 28 weeks 0 days)
Third (28 weeks 0 days until delivery)
Unspecified
Twin Pregnancy
ICD-10-CM Code/Documentation
O30.021
Conjoined twin pregnancy, first trimester
O30.022
Conjoined twin pregnancy, second trimester
O30.023
Conjoined twin pregnancy, third trimester
O30.029
Conjoined twin pregnancy, unspecified trimester
O30.011
Twin pregnancy, monochorionic/monoamniotic, first trimester
O30.012
Twin pregnancy, monochorionic/monoamniotic, second trimester
O30.013
Twin pregnancy, monochorionic/monoamniotic, third trimester
O30.019
Twin pregnancy, monochorionic/monoamniotic, unspecified trimester
O30.001
Twin pregnancy, unspecified number of placenta and unspecified number of amniotic sacs, first trimester
O30.002
Twin pregnancy, unspecified number of placenta and unspecified number of amniotic sacs, second trimester
O30.003
Twin pregnancy, unspecified number of placenta and unspecified number of amniotic sacs, third trimester
O30.009
Twin pregnancy, unspecified number of placenta and unspecified number of amniotic sacs, unspecified trimester
O30.031
Twin pregnancy, monochorionic/diamniotic, first trimester
O30.032
Twin pregnancy, monochorionic/diamniotic, second trimester
O30.033
Twin pregnancy, monochorionic/diamniotic, third trimester
O30.039
Twin pregnancy, monochorionic/diamniotic, unspecified trimester
O30.041
Twin pregnancy, dichorionic/diamniotic, first trimester
O30.042
Twin pregnancy, dichorionic/diamniotic, second trimester
O30.043
Twin pregnancy, dichorionic/diamniotic, third trimester
O30.049
Twin pregnancy, dichorionic/diamniotic, unspecified trimester
O30.091
Twin pregnancy, unable to determine number of placenta and number of amniotic sacs, first trimester
O30.092
Twin pregnancy, unable to determine number of placenta and number of amniotic sacs, second trimester
O30.093
Twin pregnancy, unable to determine number of placenta and number of amniotic sacs, third trimester
O30.099
Twin pregnancy, unable to determine number of placenta and number of amniotic sacs, unspecified trimester
Documentation and Coding Example
Patient is a 30-year-old Caucasian female who is accompanied by her husband for their first prenatal visit following a positive home pregnancy test. Patient had light vaginal bleeding and breast tenderness 4 weeks ago that she thought was her period. Her breasts have remained tender, she has had nausea without vomiting in the mornings and headaches in the afternoon and evenings. PMH is unremarkable. Patient is a legal secretary, husband is an attorney. Wt. 112 lbs. HR 70, RR 12, BP 102/60. PERRL, neck supple, thyroid gland smooth with slight fullness on right side. Mucous membranes, oral mucosa moist and pink, teeth in good repair. Breath sounds clear, equal bilaterally. HR regular without gallop or rub. Peripheral pulses strong, no edema noted. Breasts full, without masses. Abdomen soft, non-distended, bowel sounds present. FHR not found, patient assured this is quite normal. Pelvic exam shows normal external genitalia. Speculum easily inserted, pap smear obtained along with cultures for Chlamydia, GC, herpes, and HPV. Vaginal mucosa is deep red, cervix closed with bluish discoloration. Bimanual exam positive for gravid retroflexed uterus. Vaginal US reveals a dichorionic/diamniotic twin gestation at 11-12 weeks. Placentas are low lying but do not appear to cross the cervix. FHR strong. No abnormal findings. Patient and husband are congratulated. Copies of US are printed for them to keep. Prescription for prenatal vitamins written. Questions answered. Counseled in diet and exercise. RTC in 3 weeks. Sooner if problems arise.
Diagnosis Code(s)
O30.041
Twin pregnancy, dichorionic/diamniotic, first trimester
Z3A.11
11 weeks gestation of pregnancy
Coding Note(s)
Carefully select the code identifying the number of placentas and amniotic sacs and note that there is a differentiation between an unspecified number and the inability to determine the number. Weeks of gestation is documented at between 11 and 12 weeks, thus the code for 11 weeks is selected from category Z3A.
Other Conditions Related to Pregnancy
There are specific codes for maternal care predominantly related to pregnancy for conditions such as low weight gain, fatigue during pregnancy, herpes gestationis, and complications of anesthesia as well as many other conditions. It should be noted that codes in category O26 are specific to pregnancy and do not include the same conditions documented as occurring in childbirth or the puerperium. Specific codes for the following conditions in pregnancy include: low weight gain (O26.1-), retained intrauterine contraceptive device (O26.3), Herpes gestationis (O26.4-), subluxation of symphysis pubis (O26.7-), exhaustion and fatigue (O26.81), and pruritic urticarial papules and plaques of pregnancy (PUPPP) (O26.86-). Codes from category O29 Complications of anesthesia during pregnancy are also specific to pregnancy and do not include complications that occur in childbirth or the puerperium.
Coding and Documentation Requirements
Identify the specific complication of pregnancy:
Anesthesia complication
Pulmonary
»Aspiration pneumonitis
»Pressure collapse of lung
»Other pulmonary complication
Cardiac
»Cardiac arrest
»Cardiac failure
»Other cardiac complication
Central nervous system
»Cerebral anoxia
»Other central nervous system complication
Toxic reaction to local anesthesia
Spinal and epidural anesthesia complication
»Spinal/epidural induced headache
»Other complication of spinal/epidural anesthesia
Failed or difficult intubation
Other complication of anesthesia
Herpes gestationis
Low weight gain
Pregnancy related exhaustion and fatigue
Pruritic urticarial papules and plaques of pregnancy
Retained intrauterine contraceptive device
Subluxation of symphysis pubis
For antepartum conditions, identify trimester:
First (less than 14 weeks 0 days)
Second (14 weeks 0 days to less than 28 weeks 0 days)
Third (28 weeks 0 days until delivery)
Unspecified
Complications of Anesthesia During Pregnancy
ICD-10-CM Code/Documentation
O29.01-
Aspiration pneumonitis due to anesthesia during pregnancy
O29.02-
Pressure collapse of lung due to anesthesia during pregnancy
O29.09-
Other pulmonary complications of anesthesia during pregnancy
O29.11-
Cardiac arrest due to anesthesia during pregnancy
O29.12-
Cardiac failure due to anesthesia during pregnancy
O29.19-
Other cardiac complications of anesthesia during pregnancy
O29.21-
Cerebral anoxia due to anesthesia during pregnancy
O29.29-
Other central nervous system complications of anesthesia during pregnancy
O29.3X-
Toxic reaction to local anesthesia during pregnancy
O29.4-
Spinal and epidural anesthesia induced headache during pregnancy
O29.5X-
Other complications of spinal and epidural anesthesia during pregnancy
O29.6-
Failed or difficult intubation for anesthesia during pregnancy
O29.8-
Other complications of anesthesia during pregnancy
O29.9-
Unspecified complication of anesthesia during pregnancy
Note: Subcategory codes require fifth or sixth characters to identify the trimester
Documentation and Coding Example
Anesthesiologist Note: Patient is a 31-year-old female who was admitted for traumatic fracture of right tibia and fibula requiring open reduction and internal fixation. Because she is 26 weeks pregnant the procedure was performed using spinal anesthesia. A post-anesthesia evaluation was requested to evaluate for post spinal headache/spinal fluid leak. Patient is examined supine with 30-degree elevation in HOB. She states she has a history of migraine headaches and the pain she experienced when she tried to get up for crutch training was worse than migraine pain. She currently feels just a dull ache toward the top of her head and in the frontal area. With HOB at 45 degrees the headache begins to worsen and at 60 degrees it is unbearable. Headache decreases when HOB is lowered. Spinal column examined with patient lying on her side. There is a needle puncture site at L4 which is consistent with anesthesia record. No redness or drainage noted.
Impression: Post-spinal puncture headache.
Plan: Administer 1 liter LR w/500 mg caffeine sodium benzoate over 1 hour followed by 1 Liter of LR over 2 hours and re-evaluate.
Diagnosis Code(s)
O29.42
Spinal and epidural anesthesia induced headache during pregnancy, second trimester
Z3A.26
26 weeks gestation of pregnancy
Coding Note(s)
The anesthesiologist is evaluating and treating only the post-anesthesia spinal headache so that is the only code required for this episode of care.
Poor Fetal Growth
Poor fetal growth has many causes and determining the reason that the fetus has not reached its full growth potential is not always possible. Factors that affect fetal growth may be of fetal, placental, or maternal origin and include genetic or congenital anomalies, placental insufficiency, poor nutrition, or smoking.
The placenta is an organ that secretes hormones and provides nutrition, waste elimination, and oxygen exchange to the developing fetus. Placental insufficiency is a complication of pregnancy in which the fetus fails to receive adequate nutrition and/or oxygen because the placenta does not develop properly or becomes damaged during the pregnancy. Risk factors for placental insufficiency include maternal diabetes, hypertension, diseases that affect blood clotting, and smoking.
There is a subcategory for poor fetal growth specifically documented as due to known or suspected placental insufficiency. There is a separate subcategory for other known or suspected poor fetal growth. Because poor fetal growth is a complication that can affect one or more fetuses in a multiple gestation pregnancy, the codes for poor fetal growth require identification of the fetus(es) affected.
Coding and Documentation Requirements
Identify known or suspected poor fetal growth as due to:
Suspected placental insufficiency
Other known or suspected poor fetal growth (light-for-dates NOS)
Identify trimester:
First (less than 14 weeks 0 days)
Second (14 weeks 0 days to less than 28 weeks 0 days)
Third (28 weeks 0 days until delivery)
Unspecified
Identify fetus affected by complication:
Fetus 1
Fetus 2
Fetus 3
Fetus 4
Fetus 5
Other fetus
Unspecified fetus/not applicable
ICD-10-CM Code/Documentation
O36.511-
Maternal care for known or suspected placental insufficiency, first trimester
O36.512-
Maternal care for known or suspected placental insufficiency, second trimester
O36.513-
Maternal care for known or suspected placental insufficiency, third trimester
O36.519-
Maternal care for known or suspected placental insufficiency, unspecified trimester
O36.591-
Maternal care for other known or suspected poor fetal growth, first trimester
O36.592-
Maternal care for other known or suspected poor fetal growth, second trimester
O36.593-
Maternal care for other known or suspected poor fetal growth, third trimester
O36.599-
Maternal care for other known or suspected poor fetal growth, unspecified trimester
Assign 7th character for fetus affected by poor fetal growth:
1 for fetus 1
2 for fetus 2
3 for fetus 3
4 for fetus 4
5 for fetus 5
9 for other fetus
0 for unspecified fetus/not applicable
Documentation and Coding Example
Patient is a 33-year-old primipara with a triplet gestation at 27 weeks who presents for NST/BPP with perinatologist. Wt. 148, a 4 lb. wt. gain in 2 weeks. HR 88, RR 12. US verifies a triplet gestation with Fetus 1 and Fetus 2 sharing a single placenta and separate amniotic sacs. Fetus 1 is a 660-gram female with a crown to heel length of 31 cm. She is active with good FHR variability. Fetus 2 is a 720-gram female with a crown to heel length of 33 cm. She is also active with good FHR variability. Amniotic fluid is adequate. Placenta is high on the right side of the uterus. Fetus 3 is a male, approximately 360 grams and 26.5 cm in length. There is very little amniotic fluid in the sac. Placenta is small in size. FHR is 160 with no variability and very little fetal movement noted. Patient is scheduled for admission to antepartum unit for observation and monitoring. She is cautioned that the findings today are not favorable for Fetus 3 and delivery of all 3 babies may be necessary in the next day or two. Betamethasone IM is administered to stimulate fetal lung maturity. Patient’s husband is here, questions answered. Perinatology fellow will accompany family to floor.
Impression: Triplet gestation at 27 weeks with poor fetal growth, Fetus 3, due to placental insufficiency. Normal fetal growth for Fetus 1 and Fetus 2.
Diagnosis Code(s)
O36.5123
Maternal care for known or suspected placental insufficiency, second trimester, fetus 3
O41.02X3
Oligohydramnios, second trimester, fetus 3
O30.112
Triplet pregnancy with two or more monochorionic fetuses
Z3A.27
27 weeks gestation of pregnancy
Coding Note(s)
The 7th character identifies the fetus affected by the condition or complication, not the number of fetuses affected. In a single gestation, the 7th character is always ‘0’ for not applicable. The 7th character 0 would also be used for multiple gestations when the fetus affected is not specified. In this case, only one fetus is affected by the conditions and this fetus has been identified on the ultrasound as Fetus 3, so the codes for placental insufficiency and oligohydramnios are assigned the 7th character ‘3’. If more than one fetus were affected, the codes for each condition would be listed again for each identified fetus affected.
Rhesus/Other Isoimmunization
Isoimmunization of the red blood cells results when maternal antibodies cross the placenta and target fetal red blood cell antigens. This causes destruction of the fetal red blood cells and fetal anemia. Isoimmunization requires a sensitization event in the mother which is an initial exposure to the foreign red blood cell antigens. This usually occurs in a previous pregnancy during delivery when fetal red blood cells containing antigens that are not present in the mother cross into the mother’s circulation causing the mother to develop antibodies to these foreign antigens. A subsequent pregnancy in which the fetus has the same foreign antigens can significantly increase the number of antibodies in the mother’s red blood cells. The maternal antibodies may then cross the placenta and target and destroy the fetal red blood cells. While the condition primarily affects the fetus, additional maternal care is required to manage the hemolytic disease in the fetus. The most common type of incompatibility is anti-D, also known as Rh factor. However, there are other types of isoimmunization.
Isoimmunization is classified as anti-D [Rh] antibodies, also known as Rh incompatibility (O36.01-), other rhesus isoimmunization (O36.09-), anti-A sensitization (O36.11-), and other isoimmunization (O36.19-).
Coding and Documentation Requirements
Identify isoimmunization:
Rhesus
Anti-D [Rh]
Other rhesus isoimmunization
Anti-A sensitization
Other isoimmunization
Identify trimester:
First (less than 14 weeks 0 days)
Second (14 weeks 0 days to less than 28 weeks 0 days)
Third (28 weeks 0 days until delivery)
Unspecified
Identify fetus affected by complication:
Fetus 1
Fetus 2
Fetus 3
Fetus 4
Fetus 5
Other fetus
Unspecified fetus/not applicable
Maternal Isoimmunization
ICD-10-CM Code/Documentation
O36.091-
Maternal care for other rhesus isoimmunization, first trimester
O36.092-
Maternal care for other rhesus isoimmunization, second trimester
O36.093-
Maternal care for other rhesus isoimmunization, third trimester
O36.099-
Maternal care for other rhesus isoimmunization, unspecified trimester
O36.011-
Maternal care for anti-D [Rh] antibodies, first trimester
O36.012-
Maternal care for anti-D [Rh] antibodies, second trimester
O36.013-
Maternal care for anti-D [Rh] antibodies, third trimester
O36.019-
Maternal care for anti-D [Rh] antibodies, unspecified trimester
O36.191-
Maternal care for other isoimmunization, first trimester
O36.192-
Maternal care for other isoimmunization, second trimester
O36.193-
Maternal care for other isoimmunization, third trimester
O36.199-
Maternal care for other isoimmunization, unspecified trimester
O36.111-
Maternal care for Anti-A sensitization, first trimester
O36.112-
Maternal care for Anti-A sensitization, second trimester
O36.113-
Maternal care for Anti-A sensitization, third trimester
O36.119-
Maternal care for Anti-A sensitization, unspecified trimester
Documentation and Coding Example
Thirty-one-year-old G4 P1 1 1 1 Black female is being followed for known anti-U isoimmunization. Patient has an eight-year-old daughter delivered via SVD at 38 weeks. Other obstetric history includes a spontaneous abortion at 14 weeks and a stillbirth at 32 weeks at which time the anti-U isoimmunization was identified. Anti-U serum titer has been monitored throughout the pregnancy. She is now at 30 weeks and anti-U serum titer is elevated at 1:1025. She is seen today to discuss the results of her blood test. The need to monitor amniotic fluid bilirubin using serial amniocentesis so that hemolytic anemia in the fetus can be evaluated and treated if needed is also discussed. Her first amniocentesis is scheduled for tomorrow and amniocentesis will be repeated at 2-week intervals. She is advised that she may need to be delivered early depending on the results of the amniocentesis.
Diagnosis Code(s)
O36.1930
Maternal care for other isoimmunization, third trimester, fetus not applicable or unspecified
Z3A.30
30 weeks gestation of pregnancy
Coding Note(s)
Category O36 requires a 7th character extension for identification of the fetus, which only applies for multiple gestations. Since this is a single gestation, the 7th character extension is ‘0’ which indicates that identification of the affected fetus is not applicable.
Summary
Assigning the most specific codes for conditions and complications of pregnancy, childbirth, and the puerperium requires detail about the specific condition or complication. For example, patients with pre-existing diabetes mellitus must have the type of diabetes identified to capture the most specific code in Chapter 15. The severity of pre-eclampsia must be specified as mild, moderate, severe, or HELLP syndrome. Genitourinary tract infections occurring in pregnancy are specific to site. For conditions or complications that occur during pregnancy, trimester is an element of the majority of codes. For conditions and complications associated with multiple gestation that can affect one or more fetuses, the fetus experiencing the condition or complication must be identified. Current documentation analysis should include a review of all frequently encountered conditions and complications seen in pregnancy, childbirth, and the puerperium to ensure that documentation for these conditions and complications contains sufficient detail to code to the highest level of specificity.
Resources
Documentation checklists are available in Appendix A for the following condition(s):
Gestational diabetes
Multiple Gestation
Chapter 13 Quiz
1.Assignment of the most specific code for pre-existing essential hypertension (O10.0-) complicating pregnancy in Chapter 15 requires documentation of what information?
a.Identification of the specific type of hypertension as benign or malignant
b.Identification of the trimester
c.Documentation of elevated blood pressure
d.All of the above
2.A code from categories O35 or O36 is assigned only when the documentation indicates that the fetal condition is actually responsible for modifying the management of the mother. Which of the following would not support assignment of a code from these two categories?
a.Performance of diagnostic studies
b.Additional observation or special care of the mother
c.Documentation that the patient is an elderly primigravida with an increased risk for chromosomal abnormality
d.Termination of the pregnancy
3.Which of the following code categories does not require a 7th character extension to identify the fetus:
a.O30 Multiple gestation
b.O31 Complications specific to multiple gestation
c.O40 Polyhydramnios
d.O69 Labor and delivery complicated by umbilical cord complications
4.Documentation for a pregnant patient with gestational diabetes indicates that the condition is being controlled with diet and insulin. How is this coded?
a.The code for diet-controlled gestational diabetes mellitus (O24.410) is assigned along with the code for long-term (current) use of insulin (Z79.4)
b.Two codes from subcategory O24.4 are reported, one for diet-controlled gestational diabetes (O24.410) and one for insulin-controlled (O24.414)
c.Only the code for the insulin-controlled gestational diabetes is reported (O24.414)
d.The physician should be queried and the code assigned based on the response of the query
5.Documentation of peripartum cardiomyopathy is NOT reported with code O90.3 when:
a.The condition complicates pregnancy in a patient with documented pre-existing heart disease
b.The cardiomyopathy is diagnosed in the third trimester in a patient without pre-existing heart disease
c.The condition develops as a result of pregnancy
d.The condition is not diagnosed until the postpartum period
6.Assign codes for a triplet pregnancy, two monochorionic fetuses, with documented poor fetal growth due to placental insufficiency, second trimester, Fetus 2 and Fetus 3, and normal growth of Fetus 1.
a.O30.112, O36.5122
b.O30.112, O36.5120, O36.5122, O365123
c.O30.112, O36.5122, O36.5123
d.O30.112, O36.5129
7.What statement is true about antepartum hemorrhage with coagulation defects?
a.A diagnosis documented as antepartum hemorrhage with afibrinogenemia is reported with the same code as premature separation of placenta with afibrinogenemia.
b.Documentation of premature separation of placenta with disseminated intravascular coagulation and antepartum hemorrhage, requires two codes, one from category O46 and a second code from category O45.
c.Documentation of premature separation of placenta with disseminated intravascular coagulation and antepartum hemorrhage is reported with a single code from category O46.
d.Documentation of premature separation of placenta with disseminated intravascular coagulation and antepartum hemorrhage is reported with a single code from category O45.
8.What information is not required to assign the most specific code for infections of the amniotic sac and membranes?
a.Trimester
b.Infectious organism
c.Fetus
d.Identification of site as placenta or membranes/amnion
9.A pregnant patient, 12 weeks gestation, has pre-existing diabetes mellitus due to documented genetic defects of insulin action. What code would be the first-listed diagnosis?
a.O24.111 Pre-existing diabetes mellitus, type 2, in pregnancy, first trimester
b.O24.811 Other pre-existing diabetes mellitus in pregnancy, first trimester
c.E13.9 Other specified diabetes mellitus without complications
d.E11.9 Type 2 diabetes mellitus without complications
10.What information is required to assign the most specific code for pre-eclampsia?
a.Trimester, Fetus, and Severity
b.Fetus and Severity
c.Trimester and Severity (mild, moderate, severe, HELLP)
d.Laboratory results
Chapter 13 Answers and Rationales
1.Assignment of the most specific code for pre-existing essential hypertension (O10.0-) complicating pregnancy in Chapter 15 requires documentation of what information?
b.Identification of the trimester
Rationale: Hypertension is not classified as benign or malignant. There is only one code for essential hypertension (I10) in Chapter 9 Diseases of the Circulatory System. Documentation of elevated blood pressure is not sufficient to assign a code of essential hypertension. The physician must specifically document that the patient has hypertension. Assigning the most specific code does require that the trimester be identified. There are also specific codes for pre-existing hypertension complicating childbirth and the puerperium.
2.A code from categories O35 or O36 is assigned only when the documentation indicates that the fetal condition is actually responsible for modifying the management of the mother. Which of the following would not support assignment of a code from these two categories?
c.Documentation that the patient is an elderly primigravida with an increased risk for chromosomal abnormality
Rationale: Documentation that the patient is an elderly primigravida with an increased risk for chromosomal abnormality without documentation of any related tests, studies, additional observation, special care, or termination of pregnancy would not support reporting a code from categories O35 or O36. Only when the management of mother is modified in some way is reporting of codes O35 or O36 justified.
3.Which of the following code categories does not require a 7th character extension to identify the fetus?
a.O30 Multiple gestation
Rationale: Category O30 Multiple gestation identifies the details of the pregnancy as being twin, triplet, quadruplet, or other specified multiple gestation along with the number of placentas and amniotic sacs, but does not require 7th characters. The 7th characters are used for complications of pregnancy that can affect one or all fetuses and allows for identification of which particular fetus is affected by each documented complication.
4.Documentation for a pregnant patient with gestational diabetes indicates that the condition is being controlled with diet and insulin. How is this coded?
c.Only the code for the insulin-controlled gestational diabetes is reported (O24.414)
Rationale: According to the Chapter 15 guidelines, if documentation indicates the gestational diabetes is being controlled with both diet and insulin, report only the code for insulin-controlled.
5.Documentation of peripartum cardiomyopathy is NOT reported with code O90.3 when:
a.The condition complicates pregnancy in a patient with documented pre-existing heart disease
Rationale: According to Chapter 15 guidelines, code O90.3 Peripartum cardiomyopathy, is not reported for patients with pre-existing heart disease. Instead, a code from subcategory O99.4, Diseases of the circulatory system complicating pregnancy, childbirth and the puerperium, would be reported.
6.Assign codes for a triplet pregnancy, two monochorionic fetuses, with documented poor fetal growth due to placental insufficiency, second trimester, Fetus 2 and Fetus 3, and normal growth of Fetus 1.
c.O30.112, O36.5122, O36.5123
Rationale: Codes for complications of multiple gestation affecting one or more fetuses are reported for each fetus affected by the complication. In this case both Fetus 2 and Fetus 3 have documented poor fetal growth due to placental insufficiency and so code O36.512- is reported twice, once with 7th character extension 2 to identify the complication affecting Fetus 2, and once with 7th character extension 3 to identify the complication affecting Fetus 3.
7.What statement is true about antepartum hemorrhage with coagulation defects?
d.Documentation of premature separation of placenta with disseminated intravascular coagulation and antepartum hemorrhage is reported with a single code from category O45.
Rationale: A combination code is reported that identifies both the premature separation of the placenta and the specific type of coagulation defect. Antepartum hemorrhage due to coagulation defects is not reported additionally because there is an excludes note under O46. When there is documentation of an antepartum hemorrhage due to coagulation defect with documentation of premature separation of the placenta, only the code from category O45 is reported.
8.What information is not required to assign the most specific code for infections of the amniotic sac and membranes?
b.Infectious organism
Rationale: To assign the most specific code from subcategory O41.1, trimester, fetus, and site of infection is required. The infectious organism is not a component of codes in subcategory O41.1.
9.A pregnant patient, 12 weeks gestation, has pre-existing diabetes mellitus due to documented genetic defects of insulin action. What code would be the first-listed diagnosis?
b.O24.811 Other pre-existing diabetes mellitus in pregnancy, first trimester
Rationale: In pregnancy, the code from Chapter 15 identifying the type of pre-existing diabetes is the first-listed diagnosis. Other pre-existing diabetes mellitus includes that specified as due to genetic defects of insulin action, so code O24.811 is the correct code.
10.What information is required to assign the most specific code for pre-eclampsia?
c.Trimester and Severity (mild, moderate, severe, HELLP)
Rationale: Trimester and severity are required to assign the most specific code. Pre-eclampsia codes do not require identification of the fetus because all fetuses would be affected by the condition. Assignment of codes requires the physician’s documentation that the patient has the condition; laboratory results are not required.